[obm-l] para os que querem sair da lista!!!!!!!!!!!!!!!!!!!!!!!!!!!!!!!!!!!!!!!!!!!!

2020-03-13 Por tôpico Mauricio de Araujo
Olha que bacana!!!
Eu coloco no google "obm-l sair" e descubro o link
http://www.mat.puc-rio.br/~nicolau/olimp/obm-l.html

Que legal!!! ele diz o que deve ser feito para sair do grupo, vejam só que
maravilha

Simples assim Ah Google, seu sabe tudo

Att,
__
Mauricio de Araujo


-- 
Esta mensagem foi verificada pelo sistema de antiv�rus e
 acredita-se estar livre de perigo.



Re: [obm-l]

2019-11-28 Por tôpico Mauricio de Araujo
A impressão que eu tenho é a de que a quantidade de termos com i zeros
passa a formar uma PA de terceira ordem.. 4 termos com 1 zero, 10 termos
com 2 zeros e assim por diante... Não consegui provar, acho que teria de
pensar mais...

S=(4,10,20,35,56,...)

Cheguei a esta ideia pensando assim:

Vamos concatenar os números x1, 1, x2, 3, x3, 9, x4 onde x1, x2, x3, x4
seriam a solução da equação x1 + x2 +x3 + x4 = Z (Z igual ao número de
zeros que quero "distribuir")... outro jeito de ver seria assim: tenho de
colocar zeros nos espaços a seguir _1_3_9_... chamei o primeiro espaço de
x1, o segundo de x2, o terceiro de x3 e o quarto de x4.

A equação x1 + x2 + x3 + x4 = 1 tem 4 soluções => 4 maneiras de escrever
com 1 zero os termos da sequencia do Joaozinho (4!/3!)

A equação x1 + x2 + x3 + x4 = 2 tem 10 soluções => 10 maneiras de escrever
com 2 zeros os termos da sequencia do Joaozinho (5!/(3!.2!))

A equação x1 + x2 + x3 + x4 = 3 tem 20 soluções => 20 maneiras de escrever
com 3 zeros os termos da sequencia do Joaozinho. (6!/(3!.3!))

e assim por diante...

Aí é encontrar no braço o termo geral da sequencia S e a soma dos seus n
primeiros termos (polinômio de grau 3)... Acho que dá trabalho mas dá para
chegar no resultado... vou ficar devendo ele :)




Att,
______
Mauricio de Araujo



Em qui., 28 de nov. de 2019 às 12:17, Jamil Silva <
jamildasi...@hotmail.com.br> escreveu:

> E se fosse:
>
> 20139, 21039, 21309, 21390, 200139, 201039, 201309, 201390, 210039,
> 210309, 210390, 213090, 213900, ...
> Qual o 2020º termo, mantendo as propriedades anteriores acrescidas do
> algarismo 3(três) em qualquer posição entre um e nove ?
>
>
>
>
>
> Enviado do Email <https://go.microsoft.com/fwlink/?LinkId=550986> para
> Windows 10
>
>
>
> --
> Esta mensagem foi verificada pelo sistema de antivírus e
> acredita-se estar livre de perigo.
>

-- 
Esta mensagem foi verificada pelo sistema de antiv�rus e
 acredita-se estar livre de perigo.



[obm-l] Re: [obm-l] Paciência de fim de ano com números:

2019-11-27 Por tôpico Mauricio de Araujo
corrigindo:  Existem 19 termos com menos de 4 zeros (3+6+10=19).


Att,
__
Mauricio de Araujo



Em qua., 27 de nov. de 2019 às 23:03, Mauricio de Araujo <
mauricio.de.ara...@gmail.com> escreveu:

> Não sei se compliquei no raciocinio mas fiz assim...
>
> Ignorando inicialmente a ordem dos termos, seja A(i) o numero de termos
> com i zeros. Não é difícil identificar a seguinte recorrência:
>
> A(i) = i+1 + A(i-1), com A(0) = 1.
>
> Temos então 3 termos com 1 zero, 6 termos com 2 zeros, 10 termos com 3
> zeros, etc.
>
> Daria então a seguinte sequência:
>
> S = (3,6,10,15,21,28,)
>
> que é uma PA de segunda ordem já que as diferenças dos termos formam um PA
> (3,4,5,6,7,...).
>
> Assim, por exemplo, o vigésimo termo da sequencia original seria escrito
> com 4 zeros e seria o primeiro dos termos com 4 zeros (obedecendo a
> ordenação): 219. Existem 19 termos com menos de 3 zeros (3+6+10=19).
>
> Para chegarmos ao termo de ordem 2020 da sequencia do Joaozinho, teremos
> de somar os termos da sequencia S até chegar próximo do termo em questão.
>
> O termo geral da PA de segunda ordem acima  é A(n) acima é = (1/2)n^2 +
> (3/2)n + 1. (a mostrar)
>
> A soma dos n primeiros termos da PA de segunda ordem acima é: S(n) =
> (1/2).n.(n+1).(2n+1)/6 + (3/2).n.(n+1)/2 + n.1. (a mostrar)
>
> Após algumas tentativas consegui chegar a n = 21. Para este n, teríamos
> escrito 2023 termos da sequencia original do Joaozinho. S(21) = 2023.
>
> Assim o termo de ordem 2023 seria 2190 (21 zeros)
>
> Logo o termo de ordem 2020 seria 21000900 (21 zeros)
>
>
>
> Att,
> __
> Mauricio de Araujo
> 
>
>
> Em ter., 26 de nov. de 2019 às 14:37, Jamil Silva 
> escreveu:
>
>> Joãozinho, entendiado com a chegado do fim do ano, começou a escrever a
>> seguinte sequência :
>>
>> 2019, 2109, 2190, 20019, 20109, 20190, 21009, 21090, 21900, 200019,
>> 200109, ...
>>
>> Começando com 2019, o sucessor de cada termo deve ter as seguintes
>> propriedades :
>>
>> I) Ser necessariamente o menor número possível maior que seu antecessor.
>>
>> II) Conter somente os algarismos de 2019 tal que necessariamente :
>>
>> a) 2 seja sempre o primeiro
>> b) os últimos sejam sempre 0 ou 9
>> c) Somente zero possa se repetir ou aparecer mais de uma vez.
>> d) o algarismo 1 esteja sempre em qualquer posição entre 2 e 9
>>
>> Calcule em que número Joãozinho deve parar até escrever os primeiros 2020
>> termos desta sequencia..
>>
>> --
>> Esta mensagem foi verificada pelo sistema de antivírus e
>>  acredita-se estar livre de perigo.
>>
>> =
>> Instru�ões para entrar na lista, sair da lista e usar a lista em
>> http://www.mat.puc-rio.br/~obmlistas/obm-l.html
>> =
>>
>

-- 
Esta mensagem foi verificada pelo sistema de antiv�rus e
 acredita-se estar livre de perigo.



[obm-l] Re: [obm-l] Paciência de fim de ano com números:

2019-11-27 Por tôpico Mauricio de Araujo
Não sei se compliquei no raciocinio mas fiz assim...

Ignorando inicialmente a ordem dos termos, seja A(i) o numero de termos com
i zeros. Não é difícil identificar a seguinte recorrência:

A(i) = i+1 + A(i-1), com A(0) = 1.

Temos então 3 termos com 1 zero, 6 termos com 2 zeros, 10 termos com 3
zeros, etc.

Daria então a seguinte sequência:

S = (3,6,10,15,21,28,)

que é uma PA de segunda ordem já que as diferenças dos termos formam um PA
(3,4,5,6,7,...).

Assim, por exemplo, o vigésimo termo da sequencia original seria escrito
com 4 zeros e seria o primeiro dos termos com 4 zeros (obedecendo a
ordenação): 219. Existem 19 termos com menos de 3 zeros (3+6+10=19).

Para chegarmos ao termo de ordem 2020 da sequencia do Joaozinho, teremos de
somar os termos da sequencia S até chegar próximo do termo em questão.

O termo geral da PA de segunda ordem acima  é A(n) acima é = (1/2)n^2 +
(3/2)n + 1. (a mostrar)

A soma dos n primeiros termos da PA de segunda ordem acima é: S(n) =
(1/2).n.(n+1).(2n+1)/6 + (3/2).n.(n+1)/2 + n.1. (a mostrar)

Após algumas tentativas consegui chegar a n = 21. Para este n, teríamos
escrito 2023 termos da sequencia original do Joaozinho. S(21) = 2023.

Assim o termo de ordem 2023 seria 2190 (21 zeros)

Logo o termo de ordem 2020 seria 21000900 (21 zeros)



Att,
__
Mauricio de Araujo



Em ter., 26 de nov. de 2019 às 14:37, Jamil Silva 
escreveu:

> Joãozinho, entendiado com a chegado do fim do ano, começou a escrever a
> seguinte sequência :
>
> 2019, 2109, 2190, 20019, 20109, 20190, 21009, 21090, 21900, 200019,
> 200109, ...
>
> Começando com 2019, o sucessor de cada termo deve ter as seguintes
> propriedades :
>
> I) Ser necessariamente o menor número possível maior que seu antecessor.
>
> II) Conter somente os algarismos de 2019 tal que necessariamente :
>
> a) 2 seja sempre o primeiro
> b) os últimos sejam sempre 0 ou 9
> c) Somente zero possa se repetir ou aparecer mais de uma vez.
> d) o algarismo 1 esteja sempre em qualquer posição entre 2 e 9
>
> Calcule em que número Joãozinho deve parar até escrever os primeiros 2020
> termos desta sequencia..
>
> --
> Esta mensagem foi verificada pelo sistema de antivírus e
>  acredita-se estar livre de perigo.
>
> =
> Instru�ões para entrar na lista, sair da lista e usar a lista em
> http://www.mat.puc-rio.br/~obmlistas/obm-l.html
> =
>

-- 
Esta mensagem foi verificada pelo sistema de antiv�rus e
 acredita-se estar livre de perigo.



Re: [obm-l] Anagramas

2019-09-26 Por tôpico Mauricio de Araujo
Bom, eu pensei assim:

Contando uma sílaba corretamente: 3 . 7! (admitindo que a sílaba funciona
"como uma letra"...) e multiplicando por 3 pois são três sílabas...
Depois tenho de descontar os casos em que o anagrama possui duas sílabas em
ordem que contei a mais no passo anterior: 3 . 5!
Por fim, tenho de adicionar o caso em que o anagrama possui as 3 sílabas em
ordem e que eu descontei a mais no passo anterior: 3!

Assim: N = 3 . 7! - 3 . 5! + 3! = 14766...

Alguém poderia validar este raciocínio?



Att,
__
Mauricio de Araujo



Em qua, 25 de set de 2019 às 17:28, Vanderlei Nemitz 
escreveu:

> Se puder, poste sua resolução.
>
> Muito obrigado!
>
> Em qua, 25 de set de 2019 15:02, Mauricio de Araujo <
> mauricio.de.ara...@gmail.com> escreveu:
>
>> boa tarde!
>> eu pensei em usar o principio da inclusão-enclusão... achei 14766...
>>
>> Att,
>> __
>> Mauricio de Araujo
>> 
>>
>>
>> Em sáb, 21 de set de 2019 às 21:25, Vanderlei Nemitz <
>> vanderma...@gmail.com> escreveu:
>>
>>> Boa noite pessoal!
>>>
>>> Resolvi uma questão e obtive como resposta 12246.
>>> Gostaria de saber se está correta.
>>>
>>> Quantos anagramas da palavra CONFIRMAR apresenta as letras de pelo menos
>>> uma sílaba da palavra original juntas e em ordem? Por exemplo, C*FIR*AMORN
>>> e *MARCONFIR* são anagramas válidos, mas FOCMRAIRN não é.
>>>
>>> Muito obrigado!
>>>
>>>
>>>
>>> --
>>> Esta mensagem foi verificada pelo sistema de antivírus e
>>> acredita-se estar livre de perigo.
>>
>>
>> --
>> Esta mensagem foi verificada pelo sistema de antivírus e
>> acredita-se estar livre de perigo.
>
>
> --
> Esta mensagem foi verificada pelo sistema de antivírus e
> acredita-se estar livre de perigo.

-- 
Esta mensagem foi verificada pelo sistema de antiv�rus e
 acredita-se estar livre de perigo.



Re: [obm-l] Anagramas

2019-09-25 Por tôpico Mauricio de Araujo
boa tarde!
eu pensei em usar o principio da inclusão-enclusão... achei 14766...

Att,
__
Mauricio de Araujo



Em sáb, 21 de set de 2019 às 21:25, Vanderlei Nemitz 
escreveu:

> Boa noite pessoal!
>
> Resolvi uma questão e obtive como resposta 12246.
> Gostaria de saber se está correta.
>
> Quantos anagramas da palavra CONFIRMAR apresenta as letras de pelo menos
> uma sílaba da palavra original juntas e em ordem? Por exemplo, C*FIR*AMORN
> e *MARCONFIR* são anagramas válidos, mas FOCMRAIRN não é.
>
> Muito obrigado!
>
>
>
> --
> Esta mensagem foi verificada pelo sistema de antivírus e
> acredita-se estar livre de perigo.

-- 
Esta mensagem foi verificada pelo sistema de antiv�rus e
 acredita-se estar livre de perigo.



Re: [obm-l] Re: Irracionalidade de pi

2019-09-04 Por tôpico Mauricio de Araujo
Olha, eu não recebi nenhum anexo... Tem certeza que é possível mandar anexo
aqui ?

Att,
__
Mauricio de Araujo



Em qua, 4 de set de 2019 às 10:00, Israel Meireles Chrisostomo <
israelmchrisost...@gmail.com> escreveu:

> Alguém por favor poderia me ajudar?
>
>
> <http://www.avg.com/email-signature?utm_medium=email_source=link_campaign=sig-email_content=webmail>
>  Livre
> de vírus. www.avg.com
> <http://www.avg.com/email-signature?utm_medium=email_source=link_campaign=sig-email_content=webmail>.
> <#m_-2553460319515081497_DAB4FAD8-2DD7-40BB-A1B8-4E2AA1F9FDF2>
>
> Em ter, 3 de set de 2019 às 17:33, Israel Meireles Chrisostomo <
> israelmchrisost...@gmail.com> escreveu:
>
>> Segue anexada a demonstração está na página 280 é a questão 96
>>
>>
>> <http://www.avg.com/email-signature?utm_medium=email_source=link_campaign=sig-email_content=webmail>
>>  Livre
>> de vírus. www.avg.com
>> <http://www.avg.com/email-signature?utm_medium=email_source=link_campaign=sig-email_content=webmail>.
>>
>> <#m_-2553460319515081497_m_-8444854098131741643_DAB4FAD8-2DD7-40BB-A1B8-4E2AA1F9FDF2>
>>
>> Em ter, 3 de set de 2019 às 17:24, Mauricio de Araujo <
>> mauricio.de.ara...@gmail.com> escreveu:
>>
>>> recebi a mensagem mas não a demonstração... se foi um anexo, não veio.
>>>
>>> Att,
>>> __
>>> Mauricio de Araujo
>>> 
>>>
>>>
>>> Em ter, 3 de set de 2019 às 17:06, Israel Meireles Chrisostomo <
>>> israelmchrisost...@gmail.com> escreveu:
>>>
>>>> Vcs do grupo receberam essa mensagem?
>>>>
>>>>
>>>> <http://www.avg.com/email-signature?utm_medium=email_source=link_campaign=sig-email_content=webmail>
>>>>  Livre
>>>> de vírus. www.avg.com
>>>> <http://www.avg.com/email-signature?utm_medium=email_source=link_campaign=sig-email_content=webmail>.
>>>>
>>>> <#m_-2553460319515081497_m_-8444854098131741643_m_6405779379066387885_m_4269581629504735598_DAB4FAD8-2DD7-40BB-A1B8-4E2AA1F9FDF2>
>>>>
>>>> Em ter, 3 de set de 2019 às 11:18, Israel Meireles Chrisostomo <
>>>> israelmchrisost...@gmail.com> escreveu:
>>>>
>>>>> Olá pessoal,eu tive uma ideia para se provar a irracionalidade de
>>>>> pi.Alguém ai se habilita a corrigir minha demonstração ? A proposta
>>>>> do problema é a seguinte:
>>>>>   Sem usar derivadas, integrais ou qualquer conteúdo que os tenham
>>>>> como princípio, mostre que π é irracional. Alguém por favor poderia
>>>>> me ajudar ?
>>>>> Desde já agradeço!!!
>>>>>
>>>>> --
>>>>> Israel Meireles Chrisostomo
>>>>>
>>>>
>>>>
>>>> --
>>>> Israel Meireles Chrisostomo
>>>>
>>>> --
>>>> Esta mensagem foi verificada pelo sistema de antivírus e
>>>> acredita-se estar livre de perigo.
>>>
>>>
>>> --
>>> Esta mensagem foi verificada pelo sistema de antivírus e
>>> acredita-se estar livre de perigo.
>>
>>
>>
>> --
>> Israel Meireles Chrisostomo
>>
>
>
> --
> Israel Meireles Chrisostomo
>
> --
> Esta mensagem foi verificada pelo sistema de antivírus e
> acredita-se estar livre de perigo.

-- 
Esta mensagem foi verificada pelo sistema de antiv�rus e
 acredita-se estar livre de perigo.



Re: [obm-l] Re: Irracionalidade de pi

2019-09-03 Por tôpico Mauricio de Araujo
recebi a mensagem mas não a demonstração... se foi um anexo, não veio.

Att,
__
Mauricio de Araujo



Em ter, 3 de set de 2019 às 17:06, Israel Meireles Chrisostomo <
israelmchrisost...@gmail.com> escreveu:

> Vcs do grupo receberam essa mensagem?
>
>
> <http://www.avg.com/email-signature?utm_medium=email_source=link_campaign=sig-email_content=webmail>
>  Livre
> de vírus. www.avg.com
> <http://www.avg.com/email-signature?utm_medium=email_source=link_campaign=sig-email_content=webmail>.
> <#m_4269581629504735598_DAB4FAD8-2DD7-40BB-A1B8-4E2AA1F9FDF2>
>
> Em ter, 3 de set de 2019 às 11:18, Israel Meireles Chrisostomo <
> israelmchrisost...@gmail.com> escreveu:
>
>> Olá pessoal,eu tive uma ideia para se provar a irracionalidade de
>> pi.Alguém ai se habilita a corrigir minha demonstração ? A proposta do
>> problema é a seguinte:
>>   Sem usar derivadas, integrais ou qualquer conteúdo que os tenham como
>> princípio, mostre que π é irracional. Alguém por favor poderia me
>> ajudar ?
>> Desde já agradeço!!!
>>
>> --
>> Israel Meireles Chrisostomo
>>
>
>
> --
> Israel Meireles Chrisostomo
>
> --
> Esta mensagem foi verificada pelo sistema de antivírus e
> acredita-se estar livre de perigo.

-- 
Esta mensagem foi verificada pelo sistema de antiv�rus e
 acredita-se estar livre de perigo.



[obm-l] Re: [obm-l] Re: [obm-l] Re: [obm-l] Re: [obm-l] Re: [obm-l] Pontuação de Campeonato

2019-06-07 Por tôpico Mauricio de Araujo
entendi, obrigado!!

Att,
__
Mauricio de Araujo



Em sex, 7 de jun de 2019 às 10:53, Ralph Teixeira 
escreveu:

> Pois é, por serem 4 rebaixados, a chave é olhar com carinho para o 17o
> time, que é o primeiro rebaixado. Por isso tem que olhar 17 times naquela
> conta!
>
> Por exemplo, o pior caso possível, em que seu time vai bem pra caramba mas
> rebaixa assim mesmo, é o caso em que tem 17 times bons (incluindo o seu) e
> 3 times horrorosos, que dão seus pontos para os outros 17 -- vide o exemplo
> que explicitei agora há pouco.
>
> Abraço, Ralph.
>
> On Fri, Jun 7, 2019 at 9:44 AM Mauricio de Araujo <
> mauricio.de.ara...@gmail.com> wrote:
>
>> Não seria, 16  ao invés de 17 a ser considerado nas contas? Afinal "4"
>> times descem...
>>
>> Att,
>> __
>> Mauricio de Araujo
>> 
>>
>>
>> Em sex, 7 de jun de 2019 às 07:44, Jeferson Almir <
>> jefersonram...@gmail.com> escreveu:
>>
>>> Valeu Ralph !!
>>> Só pra terminar eu precisaria exibir uma tabela que 33 pontos não é
>>> suficiente??? Abraço.
>>>
>>> Em sex, 7 de jun de 2019 às 00:22, Ralph Teixeira 
>>> escreveu:
>>>
>>>> RESPOSTA: 34 pontos.
>>>>
>>>> Quando o campeonato termina, os 17 melhores times jogaram 17x16/2=17x8
>>>> partidas entre si, mais 17x3 partidas com os ultimos 3 times. Assim, esses
>>>> 17 "melhores" times tem acesso a, no maximo, 17x11x3 pontos, ou seja, 33
>>>> pontos cada um na media (no maximo!).
>>>>
>>>> Isso significa que, se voce faz 34 pontos, com certeza escapa do
>>>> rebaixamento (eh impossivel que os 17 primeiros times tenham >=34 pontos
>>>> cada).
>>>>
>>>> Isto dito, com MUITO azar pode ser que algum time com 33 pontos seja
>>>> rebaixado. Basta imaginar que os 17 primeiros ganham TODOS os jogos dos 3
>>>> outros, e entre si cada um dos 17 primeiros ganha 8 e perde 8 (para mostrar
>>>> categoricamente que isso eh possivel, ponha os 17 times num circulo e
>>>> imagine que cada time ganha dos 8 mais proximos no sentido horario, e perde
>>>> dos outros 8). Entao voce tem uma tabela onde 17 times terminam com (cada
>>>> um) 11 vitorias e 8 derrotas, ou seja, 33 pontos cada, e um deles seria
>>>> rebaixado (em algum criterio de desempate).
>>>>
>>>> Abraco, Ralph.
>>>>
>>>> On Thu, Jun 6, 2019 at 10:44 PM Jeferson Almir <
>>>> jefersonram...@gmail.com> wrote:
>>>>
>>>>> Qual a pontuação mínima de um campeonato com 20 times  para que um
>>>>> time fique livre do rebaixamento( 4 últimos times  descem ) sabendo
>>>>> que cada time joga com todos os outros somente uma única vez??. E que
>>>>> vitória vale 3 pontos empate vale 1 ponto.
>>>>>
>>>>> --
>>>>> Esta mensagem foi verificada pelo sistema de antivírus e
>>>>> acredita-se estar livre de perigo.
>>>>
>>>>
>>>> --
>>>> Esta mensagem foi verificada pelo sistema de antivírus e
>>>> acredita-se estar livre de perigo.
>>>
>>>
>>> --
>>> Esta mensagem foi verificada pelo sistema de antivírus e
>>> acredita-se estar livre de perigo.
>>
>>
>> --
>> Esta mensagem foi verificada pelo sistema de antivírus e
>> acredita-se estar livre de perigo.
>
>
> --
> Esta mensagem foi verificada pelo sistema de antivírus e
> acredita-se estar livre de perigo.

-- 
Esta mensagem foi verificada pelo sistema de antiv�rus e
 acredita-se estar livre de perigo.



[obm-l] Re: [obm-l] Re: [obm-l] Re: [obm-l] Pontuação de Campeonato

2019-06-07 Por tôpico Mauricio de Araujo
Não seria, 16  ao invés de 17 a ser considerado nas contas? Afinal "4"
times descem...

Att,
__
Mauricio de Araujo



Em sex, 7 de jun de 2019 às 07:44, Jeferson Almir 
escreveu:

> Valeu Ralph !!
> Só pra terminar eu precisaria exibir uma tabela que 33 pontos não é
> suficiente??? Abraço.
>
> Em sex, 7 de jun de 2019 às 00:22, Ralph Teixeira 
> escreveu:
>
>> RESPOSTA: 34 pontos.
>>
>> Quando o campeonato termina, os 17 melhores times jogaram 17x16/2=17x8
>> partidas entre si, mais 17x3 partidas com os ultimos 3 times. Assim, esses
>> 17 "melhores" times tem acesso a, no maximo, 17x11x3 pontos, ou seja, 33
>> pontos cada um na media (no maximo!).
>>
>> Isso significa que, se voce faz 34 pontos, com certeza escapa do
>> rebaixamento (eh impossivel que os 17 primeiros times tenham >=34 pontos
>> cada).
>>
>> Isto dito, com MUITO azar pode ser que algum time com 33 pontos seja
>> rebaixado. Basta imaginar que os 17 primeiros ganham TODOS os jogos dos 3
>> outros, e entre si cada um dos 17 primeiros ganha 8 e perde 8 (para mostrar
>> categoricamente que isso eh possivel, ponha os 17 times num circulo e
>> imagine que cada time ganha dos 8 mais proximos no sentido horario, e perde
>> dos outros 8). Entao voce tem uma tabela onde 17 times terminam com (cada
>> um) 11 vitorias e 8 derrotas, ou seja, 33 pontos cada, e um deles seria
>> rebaixado (em algum criterio de desempate).
>>
>> Abraco, Ralph.
>>
>> On Thu, Jun 6, 2019 at 10:44 PM Jeferson Almir 
>> wrote:
>>
>>> Qual a pontuação mínima de um campeonato com 20 times  para que um time
>>> fique livre do rebaixamento( 4 últimos times  descem ) sabendo que cada
>>> time joga com todos os outros somente uma única vez??. E que vitória vale 3
>>> pontos empate vale 1 ponto.
>>>
>>> --
>>> Esta mensagem foi verificada pelo sistema de antivírus e
>>> acredita-se estar livre de perigo.
>>
>>
>> --
>> Esta mensagem foi verificada pelo sistema de antivírus e
>> acredita-se estar livre de perigo.
>
>
> --
> Esta mensagem foi verificada pelo sistema de antivírus e
> acredita-se estar livre de perigo.

-- 
Esta mensagem foi verificada pelo sistema de antiv�rus e
 acredita-se estar livre de perigo.



[obm-l] Re: [obm-l] Permutação circular( Casais brigados)

2019-04-09 Por tôpico Mauricio de Araujo
Acho que se resolveria assim:
Em primeiro lugar descubra o total de formas de se colocar os casais
alternadamente quanto ao sexo sem a restrição de que cada homem não se
sente ao lado de sua respectiva mulher...

Depois descubra o total de formas de se colocar os casais juntos, ou seja,
cada homem sentado com sua respectiva mulher, respeitando a alternância dos
sexos..

Depois subtraia um do outro..

Att.

Em ter, 9 de abr de 2019 às 14:13, matematica10complicada <
profdouglaso.del...@gmail.com> escreveu:

> Pessoa estive pensando num belo problema:
>
> Ao colocarmos 5 casais ao redor de uma mesa, quantas arrumacoes existem em
> que mesmo sexo fiquem separados e cada homem não se sente ao lado de sua
> respectiva esposa.
>
> Qualquer ajuda é bem vinda.
>
> Abraco do
> Douglas Oliveira.
>
> --
> Esta mensagem foi verificada pelo sistema de antivírus e
> acredita-se estar livre de perigo.

-- 
Esta mensagem foi verificada pelo sistema de antiv�rus e
 acredita-se estar livre de perigo.



Re: [obm-l] livros aparentemente interessantes

2019-04-02 Por tôpico Mauricio de Araujo
Valeu, vou adquirir dois que me parecem interessantes...

Att.

Em ter, 2 de abr de 2019 às 00:33, Carlos Gomes 
escreveu:

> Olá Maurício...são todos excelentes. Tenho boa parte deles. Vale a pena o
> investimento.
>
> Abraço, Cgomes.
>
> Em seg, 1 de abr de 2019 às 20:05, Mauricio de Araujo <
> mauricio.de.ara...@gmail.com> escreveu:
>
>> Boa noite!
>>
>> Alguém conhece algum dos livros presentes no link a seguir?
>>
>> https://www.awesomemath.org/shop/
>>
>> Parecem muito bons...
>>
>> Att.
>>
>> --
>> Esta mensagem foi verificada pelo sistema de antivírus e
>> acredita-se estar livre de perigo.
>
>
> --
> Esta mensagem foi verificada pelo sistema de antivírus e
> acredita-se estar livre de perigo.

-- 
Esta mensagem foi verificada pelo sistema de antiv�rus e
 acredita-se estar livre de perigo.



[obm-l] livros aparentemente interessantes

2019-04-01 Por tôpico Mauricio de Araujo
Boa noite!

Alguém conhece algum dos livros presentes no link a seguir?

https://www.awesomemath.org/shop/

Parecem muito bons...

Att.

-- 
Esta mensagem foi verificada pelo sistema de antiv�rus e
 acredita-se estar livre de perigo.



[obm-l] Re: [obm-l] Re: [obm-l] Problema olimpíada de maio

2019-01-23 Por tôpico Mauricio de Araujo
Bela solução!! mas qual foi o teu insight? Desconfiança de que havia uma
distribuição uniforme dos restos possíveis?
Att.

Em qua, 23 de jan de 2019 às 00:47, Ralph Teixeira 
escreveu:

> Hm, tive uma ideia, confiram se funciona.
>
> Seja S o conjunto dos numeros obtidos pela permutacao dos digitos de 1 a
> 7, e seja x_i a quantidade de elementos de S que deixam resto i na divisao
> por 7 (i=0,1,2,3,4,5,6).
>
> Agora vamos fazer dois pareamentos. (Ou seja, vamos criar funcoes f,g:S->S
> tal que f(f(N))=N e g(g(N))=N para todo N em S).
>
> PRIMEIRO: No primeiro pareamento, troque cada digito x de N=abcdefg pelo
> digito 8-x obtendo o numero f(N) (por exemplo, o numero N=1574326 eh
> pareado com f(N)=7314562). Claramente, f(N) esta em S, e f(f(N))=N. Como a
> soma desses dois numeros eh N+f(N)=888, que deixa resto 1 na divisao
> por 7, temos automaticamente que:
> -- Se N mod 7 = 1, entao f(N) mod 7 = 0; em outras palavras para cada
> numero N que deixa resto 1, temos exatamente um numero f(N) que deixa resto
> 0, e vice-versa; portanto x_1=x_0.
> -- Se N mod 7 = 2, entao f(N) mod 7 = 6; portanto x_2=x_6.
> -- Analogamente, x_3=x_5.
>
> SEGUNDO: Agora, g(N) eh obtido a partir de N trocando cada digito x por
> 7-x, EXCETO O DIGITO 7 que eh mantido. Por exemplo, se N=1754326 entao
> g(N)=6723451. Claramente g(N) estah em S, e g(g(N))=N. Agora, N+g(N) mod 7
> = 777 = 0 (pois aquele 7 extra pode ser jogado fora sem alterar o resto
> modulo 7). Assim, de maneira analoga ao pareamento anterior, concluimos que:
> -- x_1=x_6; x_2=x_5; x_3=x_4.
>
> Encadeando tudo, concluimos que x_0=x_1=x_2=...=x_6. Assim, o numero
> pedido eh x_0=#(S)/7=6!.
>
> Abraco, Ralph.
>
> On Tue, Jan 22, 2019 at 9:57 PM Heitor Gama Ribeiro 
> wrote:
>
>> Consideramos todos os números de 7 dígitos que se obtém permutando de
>> todas as maneiras possíveis os dígitos de 1234567. Quantos deles são
>> divisíveis por 7?
>>
>> --
>> Esta mensagem foi verificada pelo sistema de antivírus e
>> acredita-se estar livre de perigo.
>>
>
> --
> Esta mensagem foi verificada pelo sistema de antivírus e
> acredita-se estar livre de perigo.



-- 
--
Abraços,
Mauricio de Araujo
[oɾnɐɹɐ ǝp oıɔıɹnɐɯ]

-- 
Esta mensagem foi verificada pelo sistema de antiv�rus e
 acredita-se estar livre de perigo.



[obm-l] Re: [obm-l] Re: [obm-l] Demonstração com Geometria Plana?

2018-11-23 Por tôpico Mauricio de Araujo
Alguem conseguiu finalizar a demonstração?

Em qua, 21 de nov de 2018 11:52, Vanderlei Nemitz  Hummm...
> Parece que prolongando BF e DC, que se encontram num ponto Q, E é o
> ortocentro do triângulo BDQ.
> O desenho sugere isso.
> Mas como mostrar isso?
>
> Em ter, 20 de nov de 2018 23:38, Carlos Victor  escreveu:
>
>> Oi Vanderlei,
>>
>> Uma dica : tente mostrar que o ponto E é o ortocentro de um triângulo "
>> estratégico". É muito legal que você descubra sozinho
>>
>> Abraços
>>
>> Carlos Victor
>>
>> Em 20/11/2018 17:33, Vanderlei Nemitz escreveu:
>>
>> Pessoal, o seguinte problema sai "tranquilamente" usando Geometria
>> Analítica.
>> Tentei usar Geometria Plana, mas apenas girei bastante, sem concluir.
>> Será que é possível?
>>
>> Dado um ponto P situado no prolongamento do lado AB de um quadrado ABCD,
>> traçam-se as retas PC e PD. Pelo ponto E, intersecção de BC e PD,
>> conduzimos a reta AE cuja intersecção com PC é o ponto F. Provar que BF e
>> PD são perpendiculares.
>>
>> --
>> Esta mensagem foi verificada pelo sistema de antivrus e
>> acredita-se estar livre de perigo.
>>
>>
>>
>
> --
> Esta mensagem foi verificada pelo sistema de antivírus e
> acredita-se estar livre de perigo.

-- 
Esta mensagem foi verificada pelo sistema de antiv�rus e
 acredita-se estar livre de perigo.



Re: [obm-l] dica de site

2018-08-12 Por tôpico Mauricio de Araujo
"estão", para melhorar a concordância...

Em dom, 12 de ago de 2018 às 17:31, Mauricio de Araujo <
mauricio.de.ara...@gmail.com> escreveu:

> by the way, vários dos materiais está em inglês, ufa...
>
> Em dom, 12 de ago de 2018 às 17:29, Mauricio de Araujo <
> mauricio.de.ara...@gmail.com> escreveu:
>
>> Mas claro!!! Um site com material de olimpiadas de matemática.. Sim!!
>> está em russo... mas basta usar o bom e velho botão direito do mouse do
>> chrome para traduzir para o bom e velho português..
>>
>> Em sáb, 11 de ago de 2018 às 17:02, Anderson Torres <
>> torres.anderson...@gmail.com> escreveu:
>>
>>> Em ter, 7 de ago de 2018 às 22:01, Mauricio de Araujo
>>>  escreveu:
>>> >
>>> > https://www.twirpx.com/files/mathematics/olympiad/
>>>
>>> Poderia descrever o site, para que ao menos saibamos do que se trata?
>>>
>>> >
>>> > --
>>> > --
>>> > Abraços,
>>> > Mauricio de Araujo
>>> > [oɾnɐɹɐ ǝp oıɔıɹnɐɯ]
>>> >
>>> > --
>>> > Esta mensagem foi verificada pelo sistema de antivírus e
>>> > acredita-se estar livre de perigo.
>>>
>>> --
>>> Esta mensagem foi verificada pelo sistema de antivírus e
>>>  acredita-se estar livre de perigo.
>>>
>>>
>>> =========
>>> Instru�ões para entrar na lista, sair da lista e usar a lista em
>>> http://www.mat.puc-rio.br/~obmlistas/obm-l.html
>>> =
>>>
>>
>>
>> --
>> --
>> Abraços,
>> Mauricio de Araujo
>> [oɾnɐɹɐ ǝp oıɔıɹnɐɯ]
>>
>
>
> --
> --
> Abraços,
> Mauricio de Araujo
> [oɾnɐɹɐ ǝp oıɔıɹnɐɯ]
>


-- 
--
Abraços,
Mauricio de Araujo
[oɾnɐɹɐ ǝp oıɔıɹnɐɯ]

-- 
Esta mensagem foi verificada pelo sistema de antiv�rus e
 acredita-se estar livre de perigo.



Re: [obm-l] dica de site

2018-08-12 Por tôpico Mauricio de Araujo
by the way, vários dos materiais está em inglês, ufa...

Em dom, 12 de ago de 2018 às 17:29, Mauricio de Araujo <
mauricio.de.ara...@gmail.com> escreveu:

> Mas claro!!! Um site com material de olimpiadas de matemática.. Sim!! está
> em russo... mas basta usar o bom e velho botão direito do mouse do chrome
> para traduzir para o bom e velho português..
>
> Em sáb, 11 de ago de 2018 às 17:02, Anderson Torres <
> torres.anderson...@gmail.com> escreveu:
>
>> Em ter, 7 de ago de 2018 às 22:01, Mauricio de Araujo
>>  escreveu:
>> >
>> > https://www.twirpx.com/files/mathematics/olympiad/
>>
>> Poderia descrever o site, para que ao menos saibamos do que se trata?
>>
>> >
>> > --
>> > --
>> > Abraços,
>> > Mauricio de Araujo
>> > [oɾnɐɹɐ ǝp oıɔıɹnɐɯ]
>> >
>> > --
>> > Esta mensagem foi verificada pelo sistema de antivírus e
>> > acredita-se estar livre de perigo.
>>
>> --
>> Esta mensagem foi verificada pelo sistema de antivírus e
>>  acredita-se estar livre de perigo.
>>
>>
>> =
>> Instru�ões para entrar na lista, sair da lista e usar a lista em
>> http://www.mat.puc-rio.br/~obmlistas/obm-l.html
>> =
>>
>
>
> --
> --
> Abraços,
> Mauricio de Araujo
> [oɾnɐɹɐ ǝp oıɔıɹnɐɯ]
>


-- 
--
Abraços,
Mauricio de Araujo
[oɾnɐɹɐ ǝp oıɔıɹnɐɯ]

-- 
Esta mensagem foi verificada pelo sistema de antiv�rus e
 acredita-se estar livre de perigo.



Re: [obm-l] dica de site

2018-08-12 Por tôpico Mauricio de Araujo
Mas claro!!! Um site com material de olimpiadas de matemática.. Sim!! está
em russo... mas basta usar o bom e velho botão direito do mouse do chrome
para traduzir para o bom e velho português..

Em sáb, 11 de ago de 2018 às 17:02, Anderson Torres <
torres.anderson...@gmail.com> escreveu:

> Em ter, 7 de ago de 2018 às 22:01, Mauricio de Araujo
>  escreveu:
> >
> > https://www.twirpx.com/files/mathematics/olympiad/
>
> Poderia descrever o site, para que ao menos saibamos do que se trata?
>
> >
> > --
> > ----------
> > Abraços,
> > Mauricio de Araujo
> > [oɾnɐɹɐ ǝp oıɔıɹnɐɯ]
> >
> > --
> > Esta mensagem foi verificada pelo sistema de antivírus e
> > acredita-se estar livre de perigo.
>
> --
> Esta mensagem foi verificada pelo sistema de antivírus e
>  acredita-se estar livre de perigo.
>
>
> =
> Instru�ões para entrar na lista, sair da lista e usar a lista em
> http://www.mat.puc-rio.br/~obmlistas/obm-l.html
> =========
>


-- 
--
Abraços,
Mauricio de Araujo
[oɾnɐɹɐ ǝp oıɔıɹnɐɯ]

-- 
Esta mensagem foi verificada pelo sistema de antiv�rus e
 acredita-se estar livre de perigo.



[obm-l] dica de site

2018-08-07 Por tôpico Mauricio de Araujo
https://www.twirpx.com/files/mathematics/olympiad/

-- 
--
Abraços,
Mauricio de Araujo
[oɾnɐɹɐ ǝp oıɔıɹnɐɯ]

-- 
Esta mensagem foi verificada pelo sistema de antiv�rus e
 acredita-se estar livre de perigo.



[obm-l] Re: [obm-l] Combinatória ( Semana Olímpica )

2018-06-25 Por tôpico Mauricio de Araujo
Bom dia!!

Este problema está discutido na página 52 do livro "de cuántas formas",
cujo link coloco a seguir.

https://drive.google.com/file/d/1TOu47F-UPUq9b0jr4sBwQ3I5Lnk6pxQg/view?usp=sharing

Att.
--
Abraços,
Mauricio de Araujo
[oɾnɐɹɐ ǝp oıɔıɹnɐɯ]


Em dom, 24 de jun de 2018 às 15:21, Jeferson Almir 
escreveu:

> Peço ajuda nesse problema pois estou confuso em montar uma recorrência.
>
> Uma entrada de cinema custa 5 rands. Numa fila de 2n pessoas, há
> exatamente n pessoas com notas de 5 rands e as outras n possuem notas de 10
> rands. Inicialmente o caixa do cinema está vazio. De quantas maneiras
> podemos organizar a fila de modo que o caixa sempre possa dar o troco?
>
> --
> Esta mensagem foi verificada pelo sistema de antivírus e
> acredita-se estar livre de perigo.

-- 
Esta mensagem foi verificada pelo sistema de antiv�rus e
 acredita-se estar livre de perigo.



[obm-l] Re: [obm-l] Re: [obm-l] Indicação de Livro

2018-04-28 Por tôpico Mauricio de Araujo
how to prove it =>
https://drive.google.com/file/d/1D4s5ejFGQxraGnQaCDRndmrInoq7YlQx/view?usp=sharing

se gostar compre!

Att.

--
Abraços,
Mauricio de Araujo
[oɾnɐɹɐ ǝp oıɔıɹnɐɯ]

2018-04-28 13:30 GMT-03:00 Igor Caetano Diniz <icaetanodi...@gmail.com>:

> How to prove it do author Daniel Valleman(se eu não estiver errado)
>
> Excelente livro e muito claro.
> Outro bom de ter eh o Paul Halmos, Naive set theory
>
> On Sat, Apr 28, 2018, 13:17 Luiz Antonio Rodrigues <rodrigue...@gmail.com>
> wrote:
>
>> Olá, amigos!
>> Boa tarde!
>> Ontem eu fiz uma avaliação na universidade e fui mal na prova.
>> Porque eu não estudei? Não!
>> Porque não tenho conhecimentos sólidos necessários para um curso
>> superior, principalmente de lógica e demonstrações...
>> Alguém pode me indicar um (ou mais de um) livro para eu estudar por minha
>> conta? Pode ser em Inglês...
>> Muito obrigado e um abraço!
>> Luiz
>>
>> --
>> Esta mensagem foi verificada pelo sistema de antivírus e
>> acredita-se estar livre de perigo.
>
>
> --
> Esta mensagem foi verificada pelo sistema de antivírus e
> acredita-se estar livre de perigo.
>

-- 
Esta mensagem foi verificada pelo sistema de antiv�rus e
 acredita-se estar livre de perigo.



Re: [obm-l] Perguntas pro Claudio Buffara

2018-04-11 Por tôpico Mauricio de Araujo
Fiquei tentado a responder mas, como estou sem paciência, acho que vou
deixar para lá... A resposta do Artur está de bom tom, adequada e elegante.

--
Abraços,
Mauricio de Araujo
[oɾnɐɹɐ ǝp oıɔıɹnɐɯ]

2018-04-10 13:09 GMT-03:00 Marcela Costa <marcelinhacost...@gmail.com>:

> Caros participantes da lista obm-l.
>
> Tenho seguido esta lista lendo as mensagens de fora há algum tempo e
> fiquei cismada com duas mensagens que o participante Claudio Buffara enviou
> em 23 de março ( https://www.mail-archive.com/
> obm-l@mat.puc-rio.br/msg55232.html ) e 25 de março (
> https://www.mail-archive.com/obm-l@mat.puc-rio.br/msg55196.html), a
> respeito do ensino de matemática e decidi participar.
>
> Dessa forma, tenho as seguintes perguntas pra ele:
>
> 1) O Sr. diz que produtos notáveis e fatorações são "notoriamente mal
> ensinados". O Sr. tem alguma sugestão de como ensinar melhor estes tópicos?
>
> 2) O Sr. não acha um pouco arrogante fazer uma afirmação como esta, já que
> o Sr. tem um talento claramente acima da média em matemática e pertence à
> elite dos "olímpicos"?
>
> 3) O Sr. não acha que o exibicionismo com estes problemas dificílimos
> acaba por alienar os alunos normais?
>
> 4) Qual a aplicabilidade na vida real de problemas de olimpíadas de
> matemática?
>
> Sds
>
> --
> Esta mensagem foi verificada pelo sistema de antivírus e
> acredita-se estar livre de perigo.

-- 
Esta mensagem foi verificada pelo sistema de antiv�rus e
 acredita-se estar livre de perigo.



[obm-l] Re: [obm-l] Re: [obm-l] distância constante

2018-04-01 Por tôpico Mauricio de Araujo
certo, valeu!!

--
Abraços,
Mauricio de Araujo
[oɾnɐɹɐ ǝp oıɔıɹnɐɯ]

2018-03-29 19:19 GMT-03:00 Pedro José <petroc...@gmail.com>:

> Boa noite!
>
> Corrigindo
>
> MF =NG= x e EM=FN=y e não: MF=EG= x e EM = FE = y.
>
> Saudações,
> PJMS
>
> Em 29 de março de 2018 19:06, Pedro José <petroc...@gmail.com> escreveu:
>
>> Boa noite!
>>
>> Faça o desenho conforme o problema.
>>
>> Projete o ponto E em AB e chame de M. Projete o ponto G em AB e chame de
>> N.
>>
>> Os triângulos EMF e GFM (ALA) são congruentes.
>>
>> MF=EG= x e EM = FE = y.
>>
>> BM=k= x. tg30
>> NC = l = y tg30
>>
>> k + x + y + l = a = (x+y). (1 + tg30) ==> x + y = a/(1 + tg30) ==> x+ y =
>> cte.
>>
>> A partir do ponto G trace uma paralela a BC e projete o ponto D sobre
>> essa paralela e chame-o de P. O triângulo DPG é congruente aos triângulos
>> EMF e GMF (ALA).
>> Então DP=x e como GE=y, a distância mencionada é x+y, que é constante
>> como visto anteriormente.
>>
>> Saudações,
>> PJMS
>>
>>
>>
>>
>> Em 29 de março de 2018 15:11, Mauricio de Araujo <
>> mauricio.de.ara...@gmail.com> escreveu:
>>
>>> Um quadrado DEFG eh colocado no interior de um triangulo equilátero ABC
>>> de maneira que a vértice E fique sobre o lado c, o vértice F sobre o lado a
>>> e o vértice G sobre o lado b do referido triangulo. Mostrar que a distância
>>> do vértice D do quadrado ao lado a do triângulo é constante à medida que os
>>> demais vértices do quadrado se movem sobre os lados do triângulo.
>>>
>>> Como que se prova?
>>>
>>> --
>>> Abraços,
>>> Mauricio de Araujo
>>> [oɾnɐɹɐ ǝp oıɔıɹnɐɯ]
>>>
>>> --
>>> Esta mensagem foi verificada pelo sistema de antivírus e
>>> acredita-se estar livre de perigo.
>>
>>
>>
>
> --
> Esta mensagem foi verificada pelo sistema de antivírus e
> acredita-se estar livre de perigo.
>

-- 
Esta mensagem foi verificada pelo sistema de antiv�rus e
 acredita-se estar livre de perigo.



[obm-l] distância constante

2018-03-29 Por tôpico Mauricio de Araujo
Um quadrado DEFG eh colocado no interior de um triangulo equilátero ABC de
maneira que a vértice E fique sobre o lado c, o vértice F sobre o lado a e
o vértice G sobre o lado b do referido triangulo. Mostrar que a distância
do vértice D do quadrado ao lado a do triângulo é constante à medida que os
demais vértices do quadrado se movem sobre os lados do triângulo.

Como que se prova?

--
Abraços,
Mauricio de Araujo
[oɾnɐɹɐ ǝp oıɔıɹnɐɯ]

-- 
Esta mensagem foi verificada pelo sistema de antiv�rus e
 acredita-se estar livre de perigo.



[obm-l] Re: [obm-l] Revista para olímpicos (gratuita, online)

2018-02-06 Por tôpico Mauricio de Araujo
Obrigado pela dica!!

--
Abraços,
Mauricio de Araujo
[oɾnɐɹɐ ǝp oıɔıɹnɐɯ]

2018-02-05 10:44 GMT-02:00 Tássio Naia <t...@polignu.org>:

> Salve,
>
> Gostaria de sugerir aos colegas a leitura do Archimede Mathematical
> Journal, um periódico voltado para olímpicos.
>
> http://amj-math.com/
>
> Até,
> Tássio
>
> --
> Esta mensagem foi verificada pelo sistema de antivírus e
> acredita-se estar livre de perigo.

-- 
Esta mensagem foi verificada pelo sistema de antiv�rus e
 acredita-se estar livre de perigo.



[obm-l] a quem possa interessar

2017-08-09 Por tôpico Mauricio de Araujo
https://drive.google.com/drive/folders/0B8qeUE5SqcPAWFVaM1N5anN3S2M

-- 
Esta mensagem foi verificada pelo sistema de antiv�rus e
 acredita-se estar livre de perigo.



Re: [obm-l] Dica Sobre Livros

2017-07-31 Por tôpico Mauricio de Araujo
Gostaria de receber tamném, mauricio.de.ara...@gmail.com

--
Abraços,
Mauricio de Araujo
[oɾnɐɹɐ ǝp oıɔıɹnɐɯ]


2017-07-31 9:31 GMT-03:00 vinicius raimundo <vini.raimu...@gmail.com>:

> Também tenho interesse nos anexos, se for possível mandar desde já
> agradeço
>
>
> Em domingo, 30 de julho de 2017, Nowras Ali <nowras@gmail.com>
> escreveu:
>
>> Também me interesso, por favor
>>
>> Obrigado de antemão!
>>
>> Em 30 de julho de 2017 09:15, Pedro Júnior <pedromatematic...@gmail.com>
>> escreveu:
>>
>>> Oi bom dia, gostaria do link dos livros.
>>> Também quero ;)
>>>
>>> Em 30 de jul de 2017 3:08 AM, "Lucas Colucci" <
>>> lucas.colucci.so...@gmail.com> escreveu:
>>>
>>> Tenho interesse também. lucas.colucci.so...@gmail.com
>>>
>>> Muito obrigado!
>>>
>>> 2017. júl. 30. 4:10 ezt írta ("Kelvin Anjos" <kelvinan...@gmail.com>):
>>>
>>>> Como essa lista é apenas para dúvidas e problemas da obm, te envio um
>>>> e-mail com os anexos.
>>>> Se alguém mais se interessar, basta me enviar um e-mail pedindo.
>>>>
>>>> On 29 July 2017 at 11:52, Ricardo Leão <leaoricardo...@gmail.com>
>>>> wrote:
>>>>
>>>>> Eu tenho procurado os seguintes livros:
>>>>>
>>>>> - Andreescu, T; Kedlaya, K; Zeitz, P; *Mathematical Contests
>>>>> 1995-1996: Olympiad Problems from around the world, with solutions*
>>>>> (1997)
>>>>>
>>>>> - Andreescu, T; Kedlaya, K; *Mathematical Contests 1996-1997:
>>>>> Olympiad Problems from around the world, with solutions* (1998)
>>>>>
>>>>> - Andreescu, T; Kedlaya, K; *Mathematical Contests 1997-1998:
>>>>> Olympiad Problems from around the world, with solutions* (1999)
>>>>>
>>>>> Alguém aí sabe onde eu encontro esses livros em formato físico ou
>>>>> digital???
>>>>>
>>>>> --
>>>>> Esta mensagem foi verificada pelo sistema de antivírus e
>>>>> acredita-se estar livre de perigo.
>>>>
>>>>
>>>>
>>>> --
>>>> Esta mensagem foi verificada pelo sistema de antivírus e
>>>> acredita-se estar livre de perigo.
>>>
>>>
>>> --
>>> Esta mensagem foi verificada pelo sistema de antivírus e
>>> acredita-se estar livre de perigo.
>>>
>>>
>>>
>>> --
>>> Esta mensagem foi verificada pelo sistema de antivírus e
>>> acredita-se estar livre de perigo.
>>>
>>
>>
>> --
>> Esta mensagem foi verificada pelo sistema de antivírus e
>> acredita-se estar livre de perigo.
>
>
> --
> Esta mensagem foi verificada pelo sistema de antivírus e
> acredita-se estar livre de perigo.
>

-- 
Esta mensagem foi verificada pelo sistema de antiv�rus e
 acredita-se estar livre de perigo.



Re: [obm-l] Problema da olimpiada hungara.

2017-05-23 Por tôpico Mauricio de Araujo
Obrigado!!

--
Abraços,
Mauricio de Araujo
[oɾnɐɹɐ ǝp oıɔıɹnɐɯ]


2017-05-22 21:33 GMT-03:00 Pedro José <petroc...@gmail.com>:

> Boa noite.
>
> Tentei da última vez escrever de uma forma simples, mas não deu,
> tem muitas falhas,  não vale,
>
> Na verdade, vai se formar um período a partir da anomalia do algarismo das
> dezenas que é 1 e é a única vez que ele aparece.
> Depois será formado um período 023456789, que irá valer a princípio até o
> algarismo 2012, como é formulado o problema ou o algarismo de orem 10^2011.
> Intuitivamente é bem fácil ver, mas na hora de provar é bem difícil, que
> os números têm uma forma de geração.
>
> Algarismo de ordem 10^a é =f(a) onde f é definida, com a <=2011
>
> f(x) = 1 se a=0
> f(a) =(r + 1) mod 10, r <>0 e f(a) pertence a { 1, 2, 3, 4, 5, 6, 7 ,8 ,9}
> f(a) = 0 se r= 0
>
>  onde r é o resto da divisão euclidiana de a por 9
>
> Assim para o caso 73, a =72 e r =0.
>
> Para o algarismo de ordem 10^347 ==> r= 5 ==> algarismo 6.
>
> Mas é difícil provar, pois o algarismo cuja a soma das parcelas,
> utilizando-se o algoritmo da multiplicação, inclusive com os famosos "vai
> um", "vão dois".. der um número de 4 algarismos, ele irá influenciar os
> três algarismos seguintes, tentei até por indução mas deu um ninho de se
> horrível.
> Mas por intuição é bem plausível a periodicidade, mas vou continuar
> tentando.
>
> O número terá 4203 algarismos (ou 4202, a depender do erro da função log)
>
> Saudações,
> PJMS
>
>
> Em 19 de maio de 2017 15:36, Mauricio de Araujo <
> mauricio.de.ara...@gmail.com> escreveu:
>
>> A resposta é: 0.
>>
>>
>> --
>> Abraços,
>> Mauricio de Araujo
>> [oɾnɐɹɐ ǝp oıɔıɹnɐɯ]
>>
>>
>> 2017-05-19 12:18 GMT-03:00 Jackson Sousa <jacksonlucena...@gmail.com>:
>>
>>> Onde conferimos a resposta da questão?
>>>
>>>
>>> Em 17 de maio de 2017 09:16, Bernardo Freitas Paulo da Costa <
>>> bernardo...@gmail.com> escreveu:
>>>
>>>> É bem mais fácil.  "Monte" o produto N*N como na escola.  Vai ficar um
>>>> monte de "1" em cada linha e coluna.  A 73ª coluna tem 73 "uns".
>>>> Agora, é só ver qual foi o "vai-um" da coluna anterior.  E para isso
>>>> tem que ver a anterior da anterior, mas (dica) não precisa ir muito
>>>> longe.
>>>>
>>>> Abraços,
>>>> --
>>>> Bernardo Freitas Paulo da Costa
>>>>
>>>> 2017-05-16 22:33 GMT-03:00 Anderson Torres <
>>>> torres.anderson...@gmail.com>:
>>>> > N=99...9/9 = (10^2012-1)/9
>>>> >
>>>> > 9N = 10^2012-1
>>>> > 81N^2= 10^4024-2*10^2012+1
>>>> >
>>>> > Agora tenta aplicar módulo 10^74:
>>>> >
>>>> > 81N^2= 10^4024-2*10^2012+1
>>>> >
>>>> > 81N^2=1 (mod 10^74)
>>>> >
>>>> > Agora teria que achar o "inverso" de 81 módulo 10^74, mas não parece
>>>> > fácil de cara.
>>>> >
>>>> > Outra forma seria usar alguma indução. Pelo que vi no Python, o número
>>>> > é bonitinho:
>>>> >
>>>> > 987654320987654320987654320987654320987654320987654320987654
>>>> 320987654320987654320987654320987654320987654320987654320987
>>>> 654320987654320987654320987654320987654320987654320987654320
>>>> 987654320987654320987654320987654320987654320987654320987654
>>>> 320987654320987654320987654320987654320987654320987654320987
>>>> 654320987654320987654320987654320987654320987654320987654320
>>>> 987654320987654320987654320987654320987654320987654320987654
>>>> 320987654320987654320987654320987654320987654320987654320987
>>>> 654320987654320987654320987654320987654320987654320987654320
>>>> 987654320987654320987654320987654320987654320987654320987654
>>>> 320987654320987654320987654320987654320987654320987654320987
>>>> 654320987654320987654320987654320987654320987654320987654320
>>>> 987654320987654320987654320987654320987654320987654320987654
>>>> 320987654320987654320987654320987654320987654320987654320987
>>>> 654320987654320987654320987654320987654320987654320987654320
>>>> 987654320987654320987654320987654320987654320987654320987654
>>>> 320987654320987654320987654!
>>>

Re: [obm-l] Problema da olimpiada hungara.

2017-05-19 Por tôpico Mauricio de Araujo
A resposta é: 0.


--
Abraços,
Mauricio de Araujo
[oɾnɐɹɐ ǝp oıɔıɹnɐɯ]


2017-05-19 12:18 GMT-03:00 Jackson Sousa <jacksonlucena...@gmail.com>:

> Onde conferimos a resposta da questão?
>
>
> Em 17 de maio de 2017 09:16, Bernardo Freitas Paulo da Costa <
> bernardo...@gmail.com> escreveu:
>
>> É bem mais fácil.  "Monte" o produto N*N como na escola.  Vai ficar um
>> monte de "1" em cada linha e coluna.  A 73ª coluna tem 73 "uns".
>> Agora, é só ver qual foi o "vai-um" da coluna anterior.  E para isso
>> tem que ver a anterior da anterior, mas (dica) não precisa ir muito
>> longe.
>>
>> Abraços,
>> --
>> Bernardo Freitas Paulo da Costa
>>
>> 2017-05-16 22:33 GMT-03:00 Anderson Torres <torres.anderson...@gmail.com>
>> :
>> > N=99...9/9 = (10^2012-1)/9
>> >
>> > 9N = 10^2012-1
>> > 81N^2= 10^4024-2*10^2012+1
>> >
>> > Agora tenta aplicar módulo 10^74:
>> >
>> > 81N^2= 10^4024-2*10^2012+1
>> >
>> > 81N^2=1 (mod 10^74)
>> >
>> > Agora teria que achar o "inverso" de 81 módulo 10^74, mas não parece
>> > fácil de cara.
>> >
>> > Outra forma seria usar alguma indução. Pelo que vi no Python, o número
>> > é bonitinho:
>> >
>> > 987654320987654320987654320987654320987654320987654320987654
>> 320987654320987654320987654320987654320987654320987654320987
>> 654320987654320987654320987654320987654320987654320987654320
>> 987654320987654320987654320987654320987654320987654320987654
>> 320987654320987654320987654320987654320987654320987654320987
>> 654320987654320987654320987654320987654320987654320987654320
>> 987654320987654320987654320987654320987654320987654320987654
>> 320987654320987654320987654320987654320987654320987654320987
>> 654320987654320987654320987654320987654320987654320987654320
>> 987654320987654320987654320987654320987654320987654320987654
>> 320987654320987654320987654320987654320987654320987654320987
>> 654320987654320987654320987654320987654320987654320987654320
>> 987654320987654320987654320987654320987654320987654320987654
>> 320987654320987654320987654320987654320987654320987654320987
>> 654320987654320987654320987654320987654320987654320987654320
>> 987654320987654320987654320987654320987654320987654320987654
>> 320987654320987654320987654!
>>  32!
>> >  098765432098765432098765432098765432098765432098765432098765
>> 432098765432098765432098765432098765432098765432098765432098
>> 7654320987654320987654320987654321L
>> >
>> >
>> >
>> >
>> >
>> >
>> > Em 16 de maio de 2017 16:38, Mauricio de Araujo
>> > <mauricio.de.ara...@gmail.com> escreveu:
>> >> Dado o numero N = 1...11 formado por 2012 algarismos iguais a 1,
>> qual o
>> >> algarismo que ocupa a 73a. posição a partir do algarismo das unidades
>> do
>> >> numero N^2?
>> >> --
>> >> Abraços,
>> >> Mauricio de Araujo
>> >> [oɾnɐɹɐ ǝp oıɔıɹnɐɯ]
>> >>
>>
>> --
>> Esta mensagem foi verificada pelo sistema de antivírus e
>>  acredita-se estar livre de perigo.
>>
>>
>> =
>> Instru�ões para entrar na lista, sair da lista e usar a lista em
>> http://www.mat.puc-rio.br/~obmlistas/obm-l.html
>> =
>>
>
>
> --
> Esta mensagem foi verificada pelo sistema de antivírus e
> acredita-se estar livre de perigo.
>

-- 
Esta mensagem foi verificada pelo sistema de antiv�rus e
 acredita-se estar livre de perigo.



[obm-l] Problema da olimpiada hungara.

2017-05-16 Por tôpico Mauricio de Araujo
Dado o numero N = 1...11 formado por 2012 algarismos iguais a 1, qual o
algarismo que ocupa a 73a. posição a partir do algarismo das unidades do
numero N^2?
--
Abraços,
Mauricio de Araujo
[oɾnɐɹɐ ǝp oıɔıɹnɐɯ]

-- 
Esta mensagem foi verificada pelo sistema de antiv�rus e
 acredita-se estar livre de perigo.



[obm-l] Re: [obm-l] [obm-l] Torneio de Tênis( problema de grafos)

2017-03-24 Por tôpico Mauricio de Araujo
2017-03-23 21:45 GMT-03:00 Pedro Soares <pedrosoares...@gmail.com>:

> Em um torneio de tênis com 14 jogadores


​bom dia,

ver resposta em
http://www.urantiagaia.org/educacional/matematica/combinatoria2/Aula19-MiscelaneaII.pdf
​


--
Abraços,
Mauricio de Araujo
[oɾnɐɹɐ ǝp oıɔıɹnɐɯ]

-- 
Esta mensagem foi verificada pelo sistema de antiv�rus e
 acredita-se estar livre de perigo.



Re: [obm-l] Probleminha bacana

2017-03-13 Por tôpico Mauricio de Araujo
https://brilliant.org/practice/probability-rules-problem-solving/?p=2


--
Abraços,
Mauricio de Araujo
[oɾnɐɹɐ ǝp oıɔıɹnɐɯ]


2017-03-04 11:49 GMT-03:00 Leonardo Maia <lpm...@gmail.com>:

> É um processo de Poisson disfarçado. Realmente, o tempo é contínuo e
> perguntas gerais requerem cálculo. Porém, como meias horas formam uma hora
> por um múltiplo inteiro (dois), os dados do problema permitem a solução com
> métodos discretos.
>
> A correta solução do Carlos Gomes coincide com a resposta usando o
> processo de Poisson.
>
> Leo
>
> 2017-03-04 7:26 GMT-03:00 Carlos Gomes <cgomes...@gmail.com>:
>
>> É verdade Pedro...eu também tive exatamente o mesmo sentimento que você.
>> É tipicamente um daqueles enunciados, mal enunciados. É comum alguém pensar
>> algo e escrever outra coisa! Nesses caso tento passar para o outro lado e
>> tentar imaginar o que se passava na cabeça de que criou o problema. Dessa
>> forma eu supus  que  que quando ele diz "uniforme" ele queira dizer  que
>> tem intervalos de tempos iguais a probabilidade de se pescar um peixe seja
>> a mesma. Mas você tem razão, rigorosamente o enunciado precisaria ser
>> melhor, aliás, ser posto de uma forma correta. Mas acredito fortemente que
>> era isso que se passava na cabeça de que elaborou.
>>
>> Em 3 de março de 2017 22:10, Pedro José <petroc...@gmail.com> escreveu:
>>
>>> Boa noite!
>>>
>>> Não compreendi o problema. Para mim há uma curva de distribuição de
>>> probabilidade.
>>> Portanto não há como aplicar conceito de modelo discreto. Mas sim
>>> integral.
>>> Também, não entendi o que significa probabilidade uniforme.
>>>
>>>
>>> Saudações,
>>> PJMS
>>>
>>>
>>>
>>>
>>>
>>> Em 3 de março de 2017 11:45, Carlos Gomes <cgomes...@gmail.com>
>>> escreveu:
>>>
>>>> Ola Mauricio,
>>>>
>>>> Eu pensei assim:
>>>>
>>>> seja p a probabilidade de pegar pelo menos um peixe em meia hora (que é
>>>> o aue você quer  achar!). Assim a probabilidade de nao pegar nenhum peixe
>>>> em meia hora é 1-p.
>>>>
>>>> Como a probabilidade de pegar pelo menos um peixe em uma hora é 0,64,
>>>> segue que a probabilidade de nao pegar nenhum peixe em uma hora
>>>> é1-0,64=0,36.
>>>>
>>>> Ora, mas se nao pegou um peixe em uma hora, quer dizer que nao pegou
>>>> nenhum peixe durante a primeira meia hora e tambem nao pegou nehum peixe
>>>> durante a segunda meia hora, o que ocorre com probabilidade (1-p)(1-p)
>>>>
>>>> Assim, (1-p)^2=0,36  ==> 1-p=0,60  ==> p=0,40 (=40%).
>>>>
>>>> Cgomes.
>>>>
>>>> Em 3 de março de 2017 14:28, Mauricio de Araujo <
>>>> mauricio.de.ara...@gmail.com> escreveu:
>>>>
>>>>>
>>>>> Em um determinado lago, a probabilidade de se pegar um peixe é
>>>>> uniforme e independente ao longo do tempo. Se a probabilidade de você 
>>>>> pegar
>>>>> pelo menos um peixe em uma hora é de 64%, qual é a probabilidade de você
>>>>> pegar pelo menos um peixe em meia hora?
>>>>>
>>>>> 60%
>>>>>
>>>>> 40%
>>>>>
>>>>> 80%
>>>>>
>>>>> 32%
>>>>>
>>>>>
>>>>>
>>>>> --
>>>>> Abraços,
>>>>> Mauricio de Araujo
>>>>> [oɾnɐɹɐ ǝp oıɔıɹnɐɯ]
>>>>>
>>>>>
>>>>> --
>>>>> Esta mensagem foi verificada pelo sistema de antivírus e
>>>>> acredita-se estar livre de perigo.
>>>>
>>>>
>>>>
>>>> --
>>>> Esta mensagem foi verificada pelo sistema de antivírus e
>>>> acredita-se estar livre de perigo.
>>>>
>>>
>>>
>>> --
>>> Esta mensagem foi verificada pelo sistema de antivírus e
>>> acredita-se estar livre de perigo.
>>>
>>
>>
>> --
>> Esta mensagem foi verificada pelo sistema de antivírus e
>> acredita-se estar livre de perigo.
>>
>
>
> --
> Esta mensagem foi verificada pelo sistema de antivírus e
> acredita-se estar livre de perigo.
>

-- 
Esta mensagem foi verificada pelo sistema de antiv�rus e
 acredita-se estar livre de perigo.



[obm-l] Probleminha bacana

2017-03-03 Por tôpico Mauricio de Araujo
Em um determinado lago, a probabilidade de se pegar um peixe é uniforme e
independente ao longo do tempo. Se a probabilidade de você pegar pelo menos
um peixe em uma hora é de 64%, qual é a probabilidade de você pegar pelo
menos um peixe em meia hora?

60%

40%

80%

32%



--
Abraços,
Mauricio de Araujo
[oɾnɐɹɐ ǝp oıɔıɹnɐɯ]

-- 
Esta mensagem foi verificada pelo sistema de antiv�rus e
 acredita-se estar livre de perigo.



[obm-l] qual destes é o maior?

2017-02-10 Por tôpico Mauricio de Araujo
qual destes é maior?

2^(30!) ou (2^30)!
--
Abraços,
Mauricio de Araujo
[oɾnɐɹɐ ǝp oıɔıɹnɐɯ]

-- 
Esta mensagem foi verificada pelo sistema de antiv�rus e
 acredita-se estar livre de perigo.



[obm-l] Re: [obm-l] PDF sobre OLimpíadas

2016-06-24 Por tôpico Mauricio de Araujo
Israel, muito bom este trabalho!! vou dar uma olhada e, se for o caso,
sugerirei alguma adequação... parabéns!!

Em 24 de junho de 2016 19:25, Israel Meireles Chrisostomo <
israelmchrisost...@gmail.com> escreveu:

> Olá pessoal estou compartilhado um PDF que escrevi, acrescentei vários
> problemas:
>
> http://media.wix.com/ugd/3eea37_3049c428c55948f2b8bb069834275f50.pdf
>
> Quem tiver alguma sugestão ou correção, por favor envie para o meu email,
> pois muitas pessoas podem se beneficiar com o acerto ou mesmo se prejudicar
> com o erro.
>
> Obrigado.
>
> israelmchrisost...@gmail.com
>
> --
> Esta mensagem foi verificada pelo sistema de antivírus e
> acredita-se estar livre de perigo.




-- 

Abraços,
oɾnɐɹɐ ǝp oıɔıɹnɐɯ

-- 
Esta mensagem foi verificada pelo sistema de antiv�rus e
 acredita-se estar livre de perigo.



[obm-l] Re: [obm-l] Combinatória

2016-06-14 Por tôpico Mauricio de Araujo
Para deixar claro a questão da divisão por dois:

Nossa estratégia para montar uma comissão "não válida" é escolher um
senador entre os 30, depois escolher um inimigo e depois escolher um amigo.

Imagine que escolhemos inicialmente o senador A para formar a comissão
{A,C,B} onde A é amigo de B e inimigo de C... Esta mesma comissão é
escolhida novamente porque uma das duas situações abaixo necessariamente
acontece...

Se B for amigo de C, a mesma comissão aparecerá quando a escolha começar
pelo senador C onde a comissão será {C,A,B}

Se B for inimigo de C, a mesma comissão aparecerá quando a escolha começar
pelo senador B onde a comissão será {B,C,A}.

Logo, a mesma comissão é contada duas vezes...



Em 14 de junho de 2016 20:17, Mauricio de Araujo <
mauricio.de.ara...@gmail.com> escreveu:

> Ataquemos o problema olhando o contrário do que se quer, ou seja, vendo as
> comissões onde haja um amigo e um inimigo de um senador em particular...
>
> Isso pode ser feito assim:
>
> Número de escolhas de um certo senador: 30
> Número de inimigos a escolher para compor a comissão: 6
> Número de amigos a escolher para compor a comissão: 23
>
> Logo o total de comissões onde há um amigo e um inimigo de um certo
> senador é: 30.6.23 = 4140. Entretanto, temos de dividir este número por
> dois porque a mesma comissão aparece quando o senador escolhido é o amigo
> do primeiro... Logo o total de comissões onde existe "amigos e inimigos" é
> 2070.
>
> O total de comissões é igual a C30,3 = 4060.
>
> Logo o que se quer é 4060 - 2070 = 1990.
>
> Acho que é isso
>
> Em 11 de junho de 2016 17:21, Vanderlei Nemitz <vanderma...@gmail.com>
> escreveu:
>
>> Gostaria de uma ajuda para o seguinte problema.
>> A resposta é 1990
>> Obrigado!
>>
>>
>> Em um senado, há 30 senadores. Para cada par de senadores, eles podem ser
>> amigos ou inimigos. Cada senador tem 6 inimigos. Considere comissões
>> formadas por 3 senadores. Determine o número total de comissões, cujos
>> membros são todos amigos uns dos outros ou todos inimigos uns dos outros.
>>
>>
>>
>> --
>> Esta mensagem foi verificada pelo sistema de antivírus e
>> acredita-se estar livre de perigo.
>
>
>
>
> --
>
> Abraços,
> oɾnɐɹɐ ǝp oıɔıɹnɐɯ
>
>


-- 

Abraços,
oɾnɐɹɐ ǝp oıɔıɹnɐɯ

-- 
Esta mensagem foi verificada pelo sistema de antiv�rus e
 acredita-se estar livre de perigo.



[obm-l] Re: [obm-l] Combinatória

2016-06-14 Por tôpico Mauricio de Araujo
Ataquemos o problema olhando o contrário do que se quer, ou seja, vendo as
comissões onde haja um amigo e um inimigo de um senador em particular...

Isso pode ser feito assim:

Número de escolhas de um certo senador: 30
Número de inimigos a escolher para compor a comissão: 6
Número de amigos a escolher para compor a comissão: 23

Logo o total de comissões onde há um amigo e um inimigo de um certo senador
é: 30.6.23 = 4140. Entretanto, temos de dividir este número por dois porque
a mesma comissão aparece quando o senador escolhido é o amigo do
primeiro... Logo o total de comissões onde existe "amigos e inimigos" é
2070.

O total de comissões é igual a C30,3 = 4060.

Logo o que se quer é 4060 - 2070 = 1990.

Acho que é isso

Em 11 de junho de 2016 17:21, Vanderlei Nemitz 
escreveu:

> Gostaria de uma ajuda para o seguinte problema.
> A resposta é 1990
> Obrigado!
>
>
> Em um senado, há 30 senadores. Para cada par de senadores, eles podem ser
> amigos ou inimigos. Cada senador tem 6 inimigos. Considere comissões
> formadas por 3 senadores. Determine o número total de comissões, cujos
> membros são todos amigos uns dos outros ou todos inimigos uns dos outros.
>
>
>
> --
> Esta mensagem foi verificada pelo sistema de antivírus e
> acredita-se estar livre de perigo.




-- 

Abraços,
oɾnɐɹɐ ǝp oıɔıɹnɐɯ

-- 
Esta mensagem foi verificada pelo sistema de antiv�rus e
 acredita-se estar livre de perigo.



Re: [obm-l] letras do indice

2016-05-27 Por tôpico Mauricio de Araujo
i por causa da palavra index? j por causa da proximidade com o i? eu não
sei...

Em 27 de maio de 2016 14:59,  escreveu:

> Meus amigos um aluno me perguntou pq usamos i j para índice.
>
> Alguém sabe a razão? Abraços
>
> Hermann
>
> --
> Esta mensagem foi verificada pelo sistema de antivírus e
> acredita-se estar livre de perigo.
>



-- 

Abraços,
oɾnɐɹɐ ǝp oıɔıɹnɐɯ

-- 
Esta mensagem foi verificada pelo sistema de antiv�rus e
 acredita-se estar livre de perigo.



Re: [obm-l] Livro do Fomin

2016-04-21 Por tôpico Mauricio de Araujo
obrigado amigo mas estes links eu já tinha visto.. este livro usado por 777
dólares é, no mínimo, uma piada... e o outro link, do site da editora, está
com erro sempre... vou continuar na minha peregrinação..

Em 21 de abril de 2016 09:34, Anderson Torres <torres.anderson...@gmail.com>
escreveu:

> Em 20 de abril de 2016 22:01, Mauricio de Araujo
> <mauricio.de.ara...@gmail.com> escreveu:
> > Amigos estou procurando o livro "Leningrad Mathematical Olympiads
> 1987-1991
> > (Contests in Mathematics Series ; Vol. 1", mas sem sucesso até agora.. Já
> > revirei o google mas nada...
> >
> > Será que alguém saberia informar onde posso encontrá-lo para comprar ou
> > mesmo baixá-lo?
>
> Bem, tem na Amazon. Mas queria mesmo cópia digital, minha casa não
> cabe mais tanto papel!
>
> http://www.mathpropress.com/books/LMO/
>
>
> http://www.amazon.com/Leningrad-Mathematical-Olympiads-1987-1991-Mathematics/dp/096264014X
>
> >
> > --
> > Abraços, Mauricio.
> >
> >
> > --
> > Esta mensagem foi verificada pelo sistema de antivírus e
> > acredita-se estar livre de perigo.
>
> --
> Esta mensagem foi verificada pelo sistema de antivírus e
>  acredita-se estar livre de perigo.
>
>
> =
> Instru�ões para entrar na lista, sair da lista e usar a lista em
> http://www.mat.puc-rio.br/~obmlistas/obm-l.html
> =
>



-- 

Abraços,
oɾnɐɹɐ ǝp oıɔıɹnɐɯ

-- 
Esta mensagem foi verificada pelo sistema de antiv�rus e
 acredita-se estar livre de perigo.



[obm-l] Livro do Fomin

2016-04-20 Por tôpico Mauricio de Araujo
Amigos estou procurando o livro "Leningrad Mathematical Olympiads 1987-1991
(Contests in Mathematics Series ; Vol. 1", mas sem sucesso até agora.. Já
revirei o google mas nada...

Será que alguém saberia informar onde posso encontrá-lo para comprar ou
mesmo baixá-lo?

-- 
​Abraços, Mauricio.​
​

-- 
Esta mensagem foi verificada pelo sistema de antiv�rus e
 acredita-se estar livre de perigo.



[obm-l] Re: [obm-l] Re: [obm-l] Números inteiros

2016-01-25 Por tôpico Mauricio de Araujo
​Bernardo, acho que esta solução se complica por conta da imposição de
termos os valores das incógnitas A, B, C e D menores ou iguais a 5... Acho
que fica mais fácil usando a função abaixo:

f(x) = (x^3 + x^4 + x^5 + x^6 + x^7 + x^8) ^4

e então descobrindo o valor do coeficiente de x^27...​

Resposta: 56 soluções.

Em 24 de janeiro de 2016 22:50, Bernardo Freitas Paulo da Costa <
bernardo...@gmail.com> escreveu:

> 2016-01-24 22:30 GMT-02:00 marcone augusto araújo borges
> :
> > Determinar o número de soluções inteiras da equação a + b + c + d = 27
> > onde cada variável toma valores entre 3 e 8
>
> Faça a = A + 3, idem para B, C, D. Isso dá
>
> A+B+C+D = 27 - 4*3 = 15, onde A,B,C,D estão entre 0 e 5. Daqui em
> diante, o argumento de separar pedras (os 15 totais) com pauzinhos
> (para escolher quantas vão para A,B,C ou D) mata.
>
> Abraços,
> --
> Bernardo Freitas Paulo da Costa
>
> =
> Instru�ões para entrar na lista, sair da lista e usar a lista em
> http://www.mat.puc-rio.br/~obmlistas/obm-l.html
> =
>



-- 

Abraços,
oɾnɐɹɐ ǝp oıɔıɹnɐɯ


Re: [obm-l] Livros

2016-01-18 Por tôpico Mauricio de Araujo
Tenho eles em russo... vou deixar no link abaixo por 2 dias.
https://drive.google.com/folderview?id=0B-1sAhj7LSlyd3UzcTNSOWdjdzg=sharing

Em 18 de janeiro de 2016 18:48, regis barros 
escreveu:

> Boa noite Pessoal
> Verifiquei meu hd se há algum livro do suprun, mas lamento não há nenhum.
>
> Regis
>
>
> Em Segunda-feira, 18 de Janeiro de 2016 12:23, Guilherme Ribeiro <
> gprfaw...@gmail.com> escreveu:
>
>
> Também quero!
> Em 18/01/2016 03:28, "Research"  escreveu:
>
> Prezados,
> Faço minhas as palavras do Eduardo Beltrão.
> mathemat...@sapo.pt
>
> Atenciosamente,
>
> Nzinga
>
>
> On Jan 16, 28 Heisei, at 11:03 AM, e-...@ig.com.br wrote:
>
> Se alguém disponibilizar tais obras em PDF também gostaria de uma cópia.
> Se no DROPBOX, melhor ainda. Se não, segue meu e-mail:
> e-...@ig.com.br
> Desde já, agradeço!
> Eduardo Beltrão
>
>
> Em 14/01/2016 21:14, Jeferson Almir escreveu:
>
> Reintero o meu interesse por esses livros, caso alguém já obteve poderia
> disponibilizar uma pasta compartilhada no Dropbox seria uma boa ideia.
> Abraço Jeferson Almir
>
> Em quinta-feira, 14 de janeiro de 2016, Giovanni Celestre <
> ggabrie...@gmail.com> escreveu:
>
> Eu também, Por favor
> Obrigado
>
> 2016-01-14 13:01 GMT-02:00 Vanderlei Nemitz :
>
> Eu quero, Israel!
>
> Obrigado!
>
> Vanderlei
>
> Em 14 de janeiro de 2016 12:54, Israel Meireles Chrisostomo <
> israelmchrisost...@gmail.com> escreveu:
>
> *PROBLEMAS DE ALTA DIFICULDAD - 300 Problemas Resolvidos*
>
> *Métodos Alternativos para a Resolução de Equações e Inequações - 350
> Problemas Resolvidos*Eu tenho esses doi em espanhol, não é pdf, é o livro
> mesmo!Se tiver interessado mande um email para mim!
>
> Em 14 de janeiro de 2016 11:07, benedito freire 
> escreveu:
>
> Por favor, escreva o nome do autor completo.Talvez eu
> possa conseguir
> --
> De: Vanderlei Nemitz
> Enviada em: ‎14/‎01/‎2016 11:12
> Para: OBM
> Assunto: Re: [obm-l] Livros
>
> Você tem algum deles, Regis? Eu tinha o PDF de dois deles, em Russo, mas o
> pendrive estragou e perdi :(
>
> Em 14 de janeiro de 2016 11:01, Jefferson Cândido 
> escreveu:
>
> Muito bom! Se puder mandar também para meu e-mail, jjjeffer...@gmail.com,
> agradeço!
>
> Em 13 de janeiro de 2016 21:45, Vanderlei Nemitz 
> escreveu:
>
> *PROBLEMAS DE ALTA DIFICULDAD - 300 Problemas Resolvidos*
> *Métodos de Resoluções e Demonstrações de Desigualdades - ** 367
> Problemas*
> *Métodos Alternativos para a Resolução de Equações e Inequações - 350
> Problemas Resolvidos*
>
> *Qualquer um desses já seria uma grande ajuda!*
>
> *Obrigado!*
>
> Em 13 de janeiro de 2016 21:33, regis barros 
> escreveu:
>
> Olá Vanderlei
> Quais livros do suprun você precisa?
>
> Regis
>
>
> Em Quarta-feira, 13 de Janeiro de 2016 14:35, Vanderlei Nemitz <
> vanderma...@gmail.com> escreveu:
>
>
> Boa tarde! Alguém tem os PDFs dos livros do Suprún? Pode ser até em russo
> mesmo! Ou mesmo tenha e queira vender os livros físicos? Preciso muito
> deles, mas está em falta.
>
> Obrigado!
>
>
>
>
>
> --
> É preciso amar as pessoas como se não houvesse amanhã...
>
> Jefferson Cândido -
>
>
>
>
>


-- 
Abraços

oɾnɐɹɐ ǝp oıɔıɹnɐɯ


Re: [obm-l] Probabilidades com Dados

2015-12-03 Por tôpico Mauricio de Araujo
Em 3 de dezembro de 2015 14:37, arkon  escreveu:

> Em um jogo com três dados não-viciados


​Acho que a abordagem deve ser mais ou menos assim:

pensa no primeiro arremesso... a soma dos pontos obtidos dos três dados
pode ser 3, 4, ..., 18. Ocorre que a probabilidade dessas somas não está
distribuida uniformemente, a chance de dar 3 não é a mesma de dar 10, por
exemplo...​ isso porque só tem um jeito de dar 3 que é sair 1 em cada dado,
priobabilidade de (1/6)^3. Para sair 10 temos várias outras maneiras de
isso ocorrer... uma maneira de achar isso seria fazer uma árvore e somar as
probablidades no final Depois vc pensa no segundo arremesso que passa a
ser dependente do resultado do primeiro... se o primeiro arremesso der soma
3 então o segundo tem de ser soma 18 e assim por diante...





-- 
Abraços

oɾnɐɹɐ ǝp oıɔıɹnɐɯ

-- 
Esta mensagem foi verificada pelo sistema de antiv�rus e
 acredita-se estar livre de perigo.



Re: [obm-l] Primo?

2015-11-24 Por tôpico Mauricio de Araujo
Só para ser chato, o primo 167 caiu do céu? rsss  (sem ofensas)

No enunciado original não é mencionado o primo 167...

Em 24 de novembro de 2015 16:48, Matheus Secco 
escreveu:

> Acredito que você possa usar resíduos quadráticos:
>
> (2 legendre p) = (-1)^(p^2-1)/8
>
> (2 legendre p) == 2^(p-1)/2 (mód p)
>
> Para p = 167, temos que (167^2-1)/8 é par. Logo (2 legendre 167) = 1.
> Com isso, obtemos que 2^83 == 1 (mód 167).
>
> Abraços
>
> 2015-11-24 10:16 GMT-02:00 Pacini Bores :
>
>>
>>
>>
>> Olá Marcone,
>>
>> Observe que 2^166-1 é divisível por  167; logo   um dos fatores de
>> (2^83-1)(2^83+1) divide 167, já que 167 é primo. Só estou tentando provar
>> que é 2^83-1, que ainda não consegui.
>>
>> Pacini
>>
>> Em 24/11/2015 7:32, marcone augusto araújo borges escreveu:
>>
>> Mostre que 2^83 - 1 não é primo
>>
>> --
>> Esta mensagem foi verificada pelo sistema de antivírus e
>> acredita-se estar livre de perigo.
>>
>>
>> --
>> Esta mensagem foi verificada pelo sistema de antivírus e
>> acredita-se estar livre de perigo.
>>
>
>
> --
> Esta mensagem foi verificada pelo sistema de antivírus e
> acredita-se estar livre de perigo.
>



-- 
Abraços

oɾnɐɹɐ ǝp oıɔıɹnɐɯ

-- 
Esta mensagem foi verificada pelo sistema de antiv�rus e
 acredita-se estar livre de perigo.



Re: [obm-l] Primo?

2015-11-24 Por tôpico Mauricio de Araujo
 reposta correta mas que o professor vai relutar em aceitar...

Em 24 de novembro de 2015 09:39, Ralph Teixeira 
escreveu:

> Deixa eu ver 2^83-1 2^83-1... Ah, é, 2^83-1... Se eu me lembro
> bem, vale:
>
> 2^83-1 = 167×57912614113275649087721
>
> Confere aí se eu errei algum dígito.
>
> ;) ;) ;) ;)
>
> 2015-11-24 7:32 GMT-02:00 marcone augusto araújo borges <
> marconeborge...@hotmail.com>:
>
>> Mostre que 2^83 - 1 não é primo
>>
>> --
>> Esta mensagem foi verificada pelo sistema de antivírus e
>> acredita-se estar livre de perigo.
>>
>
>
> --
> Esta mensagem foi verificada pelo sistema de antivírus e
> acredita-se estar livre de perigo.
>



-- 
Abraços

oɾnɐɹɐ ǝp oıɔıɹnɐɯ

-- 
Esta mensagem foi verificada pelo sistema de antiv�rus e
 acredita-se estar livre de perigo.



[obm-l] site com problema bacanas para quem quiser.

2015-11-06 Por tôpico Mauricio de Araujo
http://www.math.olympiaadid.ut.ee/eng/html/index.php?id=bw


-- 
Abraços

oɾnɐɹɐ ǝp oıɔıɹnɐɯ

-- 
Esta mensagem foi verificada pelo sistema de antiv�rus e
 acredita-se estar livre de perigo.



[obm-l] questão de treinamento olimpica.

2015-10-07 Por tôpico Mauricio de Araujo
Sejam a1, a2, ..., a11 e b1, b2, ..., b11 duas permutações dos inteiros 1,
2, ..., 11. Considere os números a1.b1, a2.b2, a3.b3, ..., a11.b11. Mostre
que pelo menos dois destes números deixam o mesmo resto quando divididos
por 11.
Sugestão: Redução ao absurdo.

-- 
Abraços

oɾnɐɹɐ ǝp oıɔıɹnɐɯ

-- 
Esta mensagem foi verificada pelo sistema de antiv�rus e
 acredita-se estar livre de perigo.



[obm-l] Pergunta que gera debates

2015-09-24 Por tôpico Mauricio de Araujo
Perguntinha que gera debates, rsss

Qual o resultado de

sqrt(-4).sqrt(-9)?

6 ou -6?

​
​

​Evaluate:

​

Abraços

oɾnɐɹɐ ǝp oıɔıɹnɐɯ

-- 
Esta mensagem foi verificada pelo sistema de antiv�rus e
 acredita-se estar livre de perigo.



Re: [obm-l] Sair da lista

2015-09-18 Por tôpico Mauricio de Araujo
quer sair da lista?

veja este link:

http://www.mat.puc-rio.br/~nicolau/olimp/obm-l.html

cheguei a ele colocando no google "sair da lista obm-l"


Em 18 de setembro de 2015 15:19, jesus emanuel choquepuma <
jech...@hotmail.com> escreveu:

> Como faço pra sair da lista?
> obrigado
>
> --
> Esta mensagem foi verificada pelo sistema de antivírus e
> acredita-se estar livre de perigo.
>



-- 
Abraços

oɾnɐɹɐ ǝp oıɔıɹnɐɯ

-- 
Esta mensagem foi verificada pelo sistema de antiv�rus e
 acredita-se estar livre de perigo.



[obm-l] Re: [obm-l] Re: [obm-l] Cálculo 2 Serge Lang

2015-09-17 Por tôpico Mauricio de Araujo
Aproveitando o email do Bernardo, percebo que problemas olímpicos são o que
menos vejo por aqui... Seria interessante se mantivéssemos os propósitos da
lista. Por favor, não entenda este email como ofensivo, longe disso...



Em 16 de setembro de 2015 23:49, Bernardo Freitas Paulo da Costa <
bernardo...@gmail.com> escreveu:

> 2015-09-16 21:04 GMT-03:00 Israel Meireles Chrisostomo
> :
> >
> > Alguém sabe onde encontro na net o pdf do livro Cálculo 2 do Serge Lang?
>
> Israel, esta lista é para discutir problemas olímpicos e relacionados,
> não o que você pediu. E não sei se o Lang de Cálculo seria uma
> referência tão importante e rara assim para justificar a sua mensagem.
>
> --
> Bernardo Freitas Paulo da Costa
>
> --
> Esta mensagem foi verificada pelo sistema de antivírus e
>  acredita-se estar livre de perigo.
>
>
> =
> Instru�ões para entrar na lista, sair da lista e usar a lista em
> http://www.mat.puc-rio.br/~obmlistas/obm-l.html
> =
>



-- 
Abraços

oɾnɐɹɐ ǝp oıɔıɹnɐɯ

-- 
Esta mensagem foi verificada pelo sistema de antiv�rus e
 acredita-se estar livre de perigo.



[obm-l] Re: [obm-l] Re: [obm-l] Problema muito bacana de teoria dos números

2015-08-06 Por tôpico Mauricio de Araujo
N = 1989.

Em 6 de agosto de 2015 14:50, saulo nilson saulo.nil...@gmail.com
escreveu:

 d4-1=11
 d4=12
 d1=1
 d2=2
 d3=
 d11=(1+2+12)d8=15*17=255
 1,2,3,12,13,14,15,17,18,19,255, produto deles.

 2015-08-06 13:14 GMT-03:00 Mauricio de Araujo 
 mauricio.de.ara...@gmail.com:

 Um número natural N tem exatamente 12 divisores (incluindo 1 e N), tais
 que, colocados em ordem crescente temos d1  d2  d3  ...  d12.
 Sabe-se que o divisor que possui o índice d4 - 1 é igual ao produto (d1 +
 d2 + d4).d8. Achar N.

 --
 Abraços

 oɾnɐɹɐ ǝp oıɔıɹnɐɯ


 --
 Esta mensagem foi verificada pelo sistema de antivírus e
 acredita-se estar livre de perigo.



 --
 Esta mensagem foi verificada pelo sistema de antivírus e
 acredita-se estar livre de perigo.




-- 
Abraços

oɾnɐɹɐ ǝp oıɔıɹnɐɯ

-- 
Esta mensagem foi verificada pelo sistema de antiv�rus e
 acredita-se estar livre de perigo.



[obm-l] Problema muito bacana de teoria dos números

2015-08-06 Por tôpico Mauricio de Araujo
Um número natural N tem exatamente 12 divisores (incluindo 1 e N), tais
que, colocados em ordem crescente temos d1  d2  d3  ...  d12.
Sabe-se que o divisor que possui o índice d4 - 1 é igual ao produto (d1 +
d2 + d4).d8. Achar N.

-- 
Abraços

oɾnɐɹɐ ǝp oıɔıɹnɐɯ

-- 
Esta mensagem foi verificada pelo sistema de antiv�rus e
 acredita-se estar livre de perigo.



[obm-l] probleminha

2015-07-31 Por tôpico Mauricio de Araujo
Seja S(x) a soma dos algarismos do número natural x escrito na base 10.
Ache o menor número natural n na base 10 tal que vale a igualdade:
9.S(n) = 16.S(2n).

-- 
Abraços

oɾnɐɹɐ ǝp oıɔıɹnɐɯ

-- 
Esta mensagem foi verificada pelo sistema de antiv�rus e
 acredita-se estar livre de perigo.



Re: [obm-l] probleminha

2015-07-31 Por tôpico Mauricio de Araujo
​Pedro,

Pode ser... o peguei de uma olimpíada argentina...o enunciado original era:

​Para cada número natural x sea S(x)  la suma de sus dígitos. Hallar el
menor número natural n tal que 9S(n) = 16S(2n).

Penso que n = 0 é muito trivial mas, vai lá tudo bem, sendo rigoroso...
n0... ;)




Valeu Ralph.



Em 31 de julho de 2015 14:04, Pedro José petroc...@gmail.com escreveu:

 Bom dia!

 Não consegui compor o número. Só tinha visto para 16 e 17 algarismos e não
 achei resultado.
 Porém, o enunciado, embora claro na intenção da pergunta, não o é na
 redação:  ... *do número estritamente natural x...* ao invés de: ... *do
 número natural x*..  seria o certo.
 Uma vez que zero atente a proposição.

 x=0 == S(n)=S(2n)=0 == 9S(n) = 16S(2n)=0.

 Saudações,
 PJMS


 Em 31 de julho de 2015 12:05, Ralph Teixeira ralp...@gmail.com escreveu:

 Note:
 S(2n) eh divisivel por 9, entao
 2n eh divisivel por 9, entao
 n eh divisivel por 9, entao
 S(n) eh divisivel por 9, entao
 S(2n) eh divisivel por 81, entao
 S(n) eh divisivel por 144.

 Agora eu vou tentar arrumar algum n que satisfaz esta condicao S(n)=144 e
 S(2n)=81, para pelo menos ter uma ideia do que estah acontecendo... Mas,
 como que S(2n) eh tao menor que S(n), considerando que 2n eh maior que n?
 Ah, os digitos de 2n tem que ser bem menores que os de n...

 Entao vou fazer uma tabela com uma correspondencia entre os digitos de n
 e de 2n:
 Em 1n: 0, 1, 2, 3, 4, 5, 6, 7, 8, 9
 Em 2n: 0, 2, 4, 6, 8, 0, 2, 4, 6, 8 (se nao tiver vai um)
 Em 2n: 1, 3, 5, 7, 9, 1, 3, 5, 7, 9 (se tiver vai um da casa anterior)

 Agora:
 i) Se eu quero S(n)=144 com o menor n, eh essencial colocar poucos
 algarismos -- entao preciso de muitos algarismos grandes. Vou encher n de
 9s...
 ii) Mas ao mesmo tempo eu preciso que S(n)-S(2n)=63, isto eh, eu preciso
 que os digitos de n sejam maiores que os de 2n! Os digitos que melhor
 contribuem para este deficit sao 5 e 6, entao tambem vou encher n de
 5 e 6.
 iii) Minha estrategia de encher n de 5, 6 e 9 significa um monte de
 vai um na soma n+n... Entao vai ter que ser 5 mesmo, que dah o melhor
 deficit quando tem vai um.

 Isto me leva a tentar numeros do tipo n=A999, onde
 esse A estah ali soh para eu ajeitar a soma em 144. Claro, eu pus os 5
 antes dos 9 para o numero ficar o menor possivel.

 Mais explicitamente, supondo que sao p 5's e q 9's, eu teria:

 1n=  A55...5599...99
 2n=BC11...1199...98
 onde tecnicamente BC eh um numero de dois digitos, alias, BC=2A+1. Pus o
 A ali porque preciso de um pouco de liberdade para ajeitar n de forma que
 S(n)=144.

 Assim, S(n)=A+5p+9q e S(2n)=B+C+p+9q-1. Como eu quero S(n)-S(2n)=63,
 preciso ter 4p=63+B+C-A. Como A eh um digito, e BC=2A+1, B+C-A tem apenas
 10 hipoteses facilmente calculaveis, na ordem:

 B+C-A={1,2,3,4,5,-3,-2,-1,0,1}

 Preciso que 63+B+C-A seja multiplo de 4, e quero o menor p possivel.
 Entao vou botar B+C-A=-3, isto eh, A=6, e entao p=15. Puxa, isto tudo para
 chegar ao meu primeiro palpite para n:

 1n=065 555 555 555 555 559 999 999
 2n=131 111 111 111 111 119 999 998

 Confira que S(n)=7x9+15x5+6=144 e S(2n)=81. Hmmm, dah para botar esse 6
 num lugar melhor. Troquemos para:

 1n=055 555 555 555 555 569 999 999
 2n=111 111 111 111 111 139 999 998

 A boa noticia eh que eu jah garanto que a melhor solucao terah mesmo
 S(n)=144 e S(2n)=81. Afinal, se nao fosse isso, seria S(n)=288, que jah
 seriam 32 algarismos, e meu n ali tem bem menos do que isso.

 Agora precisamos mostrar que esse numero de 23 digitos eh o menor n
 possivel! Ou tem algum menor?

 ---///---

 Abraco, Ralph.

 2015-07-31 10:38 GMT-03:00 Alexandre Antunes 
 prof.alexandreantu...@gmail.com:


 Não dependeria da quantidade de algarismos de n?




 Atenciosamente,

 Prof. Msc. Alexandre Antunes
 www alexandre antunes com br

 Em 31 de julho de 2015 10:08, Mauricio de Araujo 
 mauricio.de.ara...@gmail.com escreveu:

 Seja S(x) a soma dos algarismos do número natural x escrito na base 10.
 Ache o menor número natural n na base 10 tal que vale a igualdade:
 9.S(n) = 16.S(2n).

 --
 Abraços

 oɾnɐɹɐ ǝp oıɔıɹnɐɯ


 --
 Esta mensagem foi verificada pelo sistema de antivírus e
 acredita-se estar livre de perigo.



 --
 Esta mensagem foi verificada pelo sistema de antivírus e
 acredita-se estar livre de perigo.



 --
 Esta mensagem foi verificada pelo sistema de antivírus e
 acredita-se estar livre de perigo.



 --
 Esta mensagem foi verificada pelo sistema de antivírus e
 acredita-se estar livre de perigo.




-- 
Abraços

oɾnɐɹɐ ǝp oıɔıɹnɐɯ

-- 
Esta mensagem foi verificada pelo sistema de antiv�rus e
 acredita-se estar livre de perigo.



[obm-l] Re: [obm-l] Questão Colégio Militar - anulada

2015-07-27 Por tôpico Mauricio de Araujo
Em 25 de julho de 2015 22:17, Martins Rama martin...@pop.com.br escreveu:

 Kacilda comparece ao trabalho julgando estar 10 minutos atrasada


​Acho que esta questão foi cancelada por conta do trecho Kacilda comparece
ao trabalho julgando estar 10 minutos atrasada​ aqui temos incluídos
os 5 minutos que ela já acredita que o relógio está atrasado ou não??



-- 
Abraços

oɾnɐɹɐ ǝp oıɔıɹnɐɯ

-- 
Esta mensagem foi verificada pelo sistema de antiv�rus e
 acredita-se estar livre de perigo.



[obm-l] Re: [obm-l] Re: [obm-l] Re: [obm-l] Questão Colégio Militar - anulada

2015-07-27 Por tôpico Mauricio de Araujo
Na verdade eu concordo com as soluções só estou atrás de um argumento
que justifique a anulação da questão... a única possibilidade que encontrei
foi alguém desenvolver um raciocínio diferente em cima da inclusão ou não
dos 5 minutos nos 10 minutos de atraso...
[]'s

Em 27 de julho de 2015 14:16, Pedro José petroc...@gmail.com escreveu:

 Boa tarde!

 Desculpe-me, mas a Kacilda, faz o julgamento com as suas premissas.
 Obviamente, se ela pensa eu o relógio está atrasado ela incluirá os 5 min.

 Sds,

 PJMS

 Em 27 de julho de 2015 13:43, Mauricio de Araujo 
 mauricio.de.ara...@gmail.com escreveu:


 Em 25 de julho de 2015 22:17, Martins Rama martin...@pop.com.br
 escreveu:

 Kacilda comparece ao trabalho julgando estar 10 minutos atrasada


 ​Acho que esta questão foi cancelada por conta do trecho Kacilda
 comparece ao trabalho julgando estar 10 minutos atrasada​ aqui temos
 incluídos os 5 minutos que ela já acredita que o relógio está atrasado ou
 não??



 --
 Abraços

 oɾnɐɹɐ ǝp oıɔıɹnɐɯ


 --
 Esta mensagem foi verificada pelo sistema de antivírus e
 acredita-se estar livre de perigo.



 --
 Esta mensagem foi verificada pelo sistema de antivírus e
 acredita-se estar livre de perigo.




-- 
Abraços

oɾnɐɹɐ ǝp oıɔıɹnɐɯ

-- 
Esta mensagem foi verificada pelo sistema de antiv�rus e
 acredita-se estar livre de perigo.



Re: [obm-l] Problema

2015-07-01 Por tôpico Mauricio de Araujo
​ou melhor, A deve evitar enquanto puder apagar algum múltiplo de 5.​

Em 1 de julho de 2015 14:21, Mauricio de Araujo 
mauricio.de.ara...@gmail.com escreveu:

 A não deve apagar nenhum múltiplo de 5.

 Em 1 de julho de 2015 14:19, Mauricio de Araujo 
 mauricio.de.ara...@gmail.com escreveu:

 ​Ao final do jogo, A terá apagado 13 números e B 12 números (para que
 sobre 2 números)... a estratégia vencedora de B seria apagar todos os
 números 3(mod5) e 4(mod5) além de 3 números 0(mod5) dos quatro existentes,
 ou seja, teria de executar 13 ações de apagar... como ele só joga 12 vezes
 A vence sempre (desde que jogue com cuidado)..​

 Em 1 de julho de 2015 13:30, Pedro José petroc...@gmail.com escreveu:

 Bom dia !
 Está errado o jogador pode escolher a sobra de E ou F antes de cabarem
 todos os números. Necessita de reanálise.
 -- Mensagem encaminhada --
 De: Pedro José petroc...@gmail.com
 Data: 1 de julho de 2015 10:54
 Assunto: Re: [obm-l] Problema
 Para: obm-l@mat.puc-rio.br



 Bom dia!


 E={1,6,11,16,21,26} e F= {4,9,14,19,24} Para qualquer par (a,b) com a Ɛ E
 e b Ɛ F == a + b ≡ 0 (mod5).
 G= {2, 7, 12, 17, 22,27} e H = {3, 8, 13, 18, 23} Para qualquer  (a,b)
 com a Ɛ G e b Ɛ H == a + b ≡ 0 (mod5).
 J= {5, 15, 20, 25} Para qualquer par (a,b) com a,b Ɛ J== a + b ≡ 0
 (mod5).

 O jogador A só ganha se restarem dois números pertencentes a J, um a G e
 outro a H, um a E e outro a F.
 Portanto o jogador B vence fácil.

 Basta para cada escolha  a do jogador A que inicia, o jogador B deve
 escolher -a | a + (-a) ≡ 0 (mod5).

 Se A escolhe em E, B escolhe em F e vice-versa.
 Se A escolhe em G, B escolhe em H e vice-versa.
 Se A escolhem J, B escolhe em J.

 Como a cardinalidade de E e G é maior que a cardinalidade de F e H e a
 cardinalidade de J é par, ao final sobrarão um elemento s Ɛ E e t Ɛ  F
 | s + t ≡ 3 (mod5)
 Saudações,
 PJMS


 Em 1 de julho de 2015 06:46, bened...@ufrnet.br escreveu:

 Problema
 Dois jogadores, A e B, disputam um jogo, em que jogam alternadamente. O
 jogador A começa. Uma jogada consiste em apagar um dos números inteiros do
 conjunto {1, 2, 3,..., 27} até que reste somente dois números. Se a soma
 desses dois últimos números for divisível por 5, o jogador A vence, caso
 contrário, vence o jogador B.
 Se cada jogador faz suas melhores jogadas, quem vence: A ou B? Qual é a
 estratégia para vencer?

 --
 Esta mensagem foi verificada pelo sistema de antivírus e
 acredita-se estar livre de perigo.




 --
 Esta mensagem foi verificada pelo sistema de antivírus e
 acredita-se estar livre de perigo.




 --
 Abraços

 oɾnɐɹɐ ǝp oıɔıɹnɐɯ




 --
 Abraços

 oɾnɐɹɐ ǝp oıɔıɹnɐɯ




-- 
Abraços

oɾnɐɹɐ ǝp oıɔıɹnɐɯ

-- 
Esta mensagem foi verificada pelo sistema de antiv�rus e
 acredita-se estar livre de perigo.



Re: [obm-l] Problema

2015-07-01 Por tôpico Mauricio de Araujo
A não deve apagar nenhum múltiplo de 5.

Em 1 de julho de 2015 14:19, Mauricio de Araujo 
mauricio.de.ara...@gmail.com escreveu:

 ​Ao final do jogo, A terá apagado 13 números e B 12 números (para que
 sobre 2 números)... a estratégia vencedora de B seria apagar todos os
 números 3(mod5) e 4(mod5) além de 3 números 0(mod5) dos quatro existentes,
 ou seja, teria de executar 13 ações de apagar... como ele só joga 12 vezes
 A vence sempre (desde que jogue com cuidado)..​

 Em 1 de julho de 2015 13:30, Pedro José petroc...@gmail.com escreveu:

 Bom dia !
 Está errado o jogador pode escolher a sobra de E ou F antes de cabarem
 todos os números. Necessita de reanálise.
 -- Mensagem encaminhada --
 De: Pedro José petroc...@gmail.com
 Data: 1 de julho de 2015 10:54
 Assunto: Re: [obm-l] Problema
 Para: obm-l@mat.puc-rio.br



 Bom dia!


 E={1,6,11,16,21,26} e F= {4,9,14,19,24} Para qualquer par (a,b) com a Ɛ E
 e b Ɛ F == a + b ≡ 0 (mod5).
 G= {2, 7, 12, 17, 22,27} e H = {3, 8, 13, 18, 23} Para qualquer  (a,b)
 com a Ɛ G e b Ɛ H == a + b ≡ 0 (mod5).
 J= {5, 15, 20, 25} Para qualquer par (a,b) com a,b Ɛ J== a + b ≡ 0
 (mod5).

 O jogador A só ganha se restarem dois números pertencentes a J, um a G e
 outro a H, um a E e outro a F.
 Portanto o jogador B vence fácil.

 Basta para cada escolha  a do jogador A que inicia, o jogador B deve
 escolher -a | a + (-a) ≡ 0 (mod5).

 Se A escolhe em E, B escolhe em F e vice-versa.
 Se A escolhe em G, B escolhe em H e vice-versa.
 Se A escolhem J, B escolhe em J.

 Como a cardinalidade de E e G é maior que a cardinalidade de F e H e a
 cardinalidade de J é par, ao final sobrarão um elemento s Ɛ E e t Ɛ  F |
 s + t ≡ 3 (mod5)
 Saudações,
 PJMS


 Em 1 de julho de 2015 06:46, bened...@ufrnet.br escreveu:

 Problema
 Dois jogadores, A e B, disputam um jogo, em que jogam alternadamente. O
 jogador A começa. Uma jogada consiste em apagar um dos números inteiros do
 conjunto {1, 2, 3,..., 27} até que reste somente dois números. Se a soma
 desses dois últimos números for divisível por 5, o jogador A vence, caso
 contrário, vence o jogador B.
 Se cada jogador faz suas melhores jogadas, quem vence: A ou B? Qual é a
 estratégia para vencer?

 --
 Esta mensagem foi verificada pelo sistema de antivírus e
 acredita-se estar livre de perigo.




 --
 Esta mensagem foi verificada pelo sistema de antivírus e
 acredita-se estar livre de perigo.




 --
 Abraços

 oɾnɐɹɐ ǝp oıɔıɹnɐɯ




-- 
Abraços

oɾnɐɹɐ ǝp oıɔıɹnɐɯ

-- 
Esta mensagem foi verificada pelo sistema de antiv�rus e
 acredita-se estar livre de perigo.



Re: [obm-l] Problema

2015-07-01 Por tôpico Mauricio de Araujo
​Ao final do jogo, A terá apagado 13 números e B 12 números (para que sobre
2 números)... a estratégia vencedora de B seria apagar todos os números
3(mod5) e 4(mod5) além de 3 números 0(mod5) dos quatro existentes, ou seja,
teria de executar 13 ações de apagar... como ele só joga 12 vezes A vence
sempre (desde que jogue com cuidado)..​

Em 1 de julho de 2015 13:30, Pedro José petroc...@gmail.com escreveu:

 Bom dia !
 Está errado o jogador pode escolher a sobra de E ou F antes de cabarem
 todos os números. Necessita de reanálise.
 -- Mensagem encaminhada --
 De: Pedro José petroc...@gmail.com
 Data: 1 de julho de 2015 10:54
 Assunto: Re: [obm-l] Problema
 Para: obm-l@mat.puc-rio.br



 Bom dia!


 E={1,6,11,16,21,26} e F= {4,9,14,19,24} Para qualquer par (a,b) com a Ɛ E
 e b Ɛ F == a + b ≡ 0 (mod5).
 G= {2, 7, 12, 17, 22,27} e H = {3, 8, 13, 18, 23} Para qualquer  (a,b) com
 a Ɛ G e b Ɛ H == a + b ≡ 0 (mod5).
 J= {5, 15, 20, 25} Para qualquer par (a,b) com a,b Ɛ J== a + b ≡ 0
 (mod5).

 O jogador A só ganha se restarem dois números pertencentes a J, um a G e
 outro a H, um a E e outro a F.
 Portanto o jogador B vence fácil.

 Basta para cada escolha  a do jogador A que inicia, o jogador B deve
 escolher -a | a + (-a) ≡ 0 (mod5).

 Se A escolhe em E, B escolhe em F e vice-versa.
 Se A escolhe em G, B escolhe em H e vice-versa.
 Se A escolhem J, B escolhe em J.

 Como a cardinalidade de E e G é maior que a cardinalidade de F e H e a
 cardinalidade de J é par, ao final sobrarão um elemento s Ɛ E e t Ɛ  F |
 s + t ≡ 3 (mod5)
 Saudações,
 PJMS


 Em 1 de julho de 2015 06:46, bened...@ufrnet.br escreveu:

 Problema
 Dois jogadores, A e B, disputam um jogo, em que jogam alternadamente. O
 jogador A começa. Uma jogada consiste em apagar um dos números inteiros do
 conjunto {1, 2, 3,..., 27} até que reste somente dois números. Se a soma
 desses dois últimos números for divisível por 5, o jogador A vence, caso
 contrário, vence o jogador B.
 Se cada jogador faz suas melhores jogadas, quem vence: A ou B? Qual é a
 estratégia para vencer?

 --
 Esta mensagem foi verificada pelo sistema de antivírus e
 acredita-se estar livre de perigo.




 --
 Esta mensagem foi verificada pelo sistema de antivírus e
 acredita-se estar livre de perigo.




-- 
Abraços

oɾnɐɹɐ ǝp oıɔıɹnɐɯ

-- 
Esta mensagem foi verificada pelo sistema de antiv�rus e
 acredita-se estar livre de perigo.



Re: [obm-l] livro

2015-06-26 Por tôpico Mauricio de Araujo
Na verdade o Esdras já colocou um link que possui o livro... aliás um ótimo
site com vários livros de nível olímpico..

Em 26 de junho de 2015 16:58, Vanderlei Nemitz vanderma...@gmail.com
escreveu:

 Também gostaria do link! Muito obrigado!

 Vanderlei

 Em 26 de junho de 2015 09:49, Mauricio de Araujo 
 mauricio.de.ara...@gmail.com escreveu:

 eu tenho, vou colocar na nuvem para vc pegar. Passo o link mais tarde;.

 Em 25 de junho de 2015 19:24, Israel Meireles Chrisostomo 
 israelmchrisost...@gmail.com escreveu:

 Olá alguém sabe onde posso encontrar um pdf para baixar do livro Wining
 Solutions de E. Lozansky. c. rousseau, se tiverem um link melhor ainda


 --
 Esta mensagem foi verificada pelo sistema de antivírus e
 acredita-se estar livre de perigo.




 --
 Abraços

 oɾnɐɹɐ ǝp oıɔıɹnɐɯ


 --
 Esta mensagem foi verificada pelo sistema de antivírus e
 acredita-se estar livre de perigo.



 --
 Esta mensagem foi verificada pelo sistema de antivírus e
 acredita-se estar livre de perigo.




-- 
Abraços

oɾnɐɹɐ ǝp oıɔıɹnɐɯ

-- 
Esta mensagem foi verificada pelo sistema de antiv�rus e
 acredita-se estar livre de perigo.



Re: [obm-l] Problema

2015-06-26 Por tôpico Mauricio de Araujo
Existe uma solução para este problema na revista Eureka no. 5.

Em 22 de junho de 2015 18:32, Douglas Oliveira de Lima 
profdouglaso.del...@gmail.com escreveu:

 Olá caros colegas, gostaria de uma ajuda no seguinte problema:

 Em uma reta há 1999 bolinhas. Algumas são verdes e as demais
 azuis(poderiam ser todas verdes ou todas azuis). Debaixo de cada bolinha
 escrevemos o número igual a soma da quantidade de bolinhas verdes à sua
 direita dela mais a quantidade de bolinhas azuis a esquerda dela. Se, na
 sequência de números assim obtida, houver exatamente três números que
 aparecem uma quantidade ímpar de vezes, quais podem ser estes números?


 Abraço


 Douglas Oliveira

 --
 Esta mensagem foi verificada pelo sistema de antivírus e
 acredita-se estar livre de perigo.




-- 
Abraços

oɾnɐɹɐ ǝp oıɔıɹnɐɯ

-- 
Esta mensagem foi verificada pelo sistema de antiv�rus e
 acredita-se estar livre de perigo.



Re: [obm-l] livro

2015-06-26 Por tôpico Mauricio de Araujo
eu tenho, vou colocar na nuvem para vc pegar. Passo o link mais tarde;.

Em 25 de junho de 2015 19:24, Israel Meireles Chrisostomo 
israelmchrisost...@gmail.com escreveu:

 Olá alguém sabe onde posso encontrar um pdf para baixar do livro Wining
 Solutions de E. Lozansky. c. rousseau, se tiverem um link melhor ainda


 --
 Esta mensagem foi verificada pelo sistema de antivírus e
 acredita-se estar livre de perigo.




-- 
Abraços

oɾnɐɹɐ ǝp oıɔıɹnɐɯ

-- 
Esta mensagem foi verificada pelo sistema de antiv�rus e
 acredita-se estar livre de perigo.



[obm-l] Re: [obm-l] Dízima

2015-06-19 Por tôpico Mauricio de Araujo
http://www.tutorbrasil.com.br/forum/matematica-olimpiadas/dizimas-periodicas-t36966.html

Em 19 de junho de 2015 11:05, Pedro Costa npc1...@gmail.com escreveu:

 Questão do livro( problemas selecionados de matemática - Gandbi- Pág.: 20
 questão : 63). Já faz dois anos que tento resolver
 este problema e não tem sucesso. Alguém de vocês poderia me ajudar.
 (questão: 63) Seja N o número de algarismos do período da dízima [image:
 \frac{1}{3^{2005}]. O número de algarismos de
 N é igual a:

 a) 952
 b) 953
 c) 954
 d) 955
 e) 956




 --
[image: Avast logo] http://www.avast.com/

 Este email foi escaneado pelo Avast antivírus.
 www.avast.com


 --
 Esta mensagem foi verificada pelo sistema de antivírus e
 acredita-se estar livre de perigo.




-- 
Abraços

oɾnɐɹɐ ǝp oıɔıɹnɐɯ

-- 
Esta mensagem foi verificada pelo sistema de antiv�rus e
 acredita-se estar livre de perigo.



Re: [obm-l] inteiros positivos

2015-05-27 Por tôpico Mauricio de Araujo
Douglas, em certo momento da sua demonstração, você diz o seguinte:

...7^x=4 (mod 9), desta forma x=2 (mod 4)...

Porém, a primeira equação é satisfeita, por exemplo, por x = 5  (7^5 - 4 é
múltiplo de 9!!!) não sendo, portanto, x côngruo a 2 mod4...

Estou errado na minha avaliação?

Em 27 de maio de 2015 10:58, Douglas Oliveira de Lima 
profdouglaso.del...@gmail.com escreveu:

 Pense que x só pode assumir 4 formas,  4k, 4k+1, 4k+2, 4k+3.
 Em 27/05/2015 10:05, Pedro José petroc...@gmail.com escreveu:

 Bom dia!

 7^3 ≡ 4*7 ≡ 1 (mod9) e não 7^3 ≡ 3*7 ≡ 1 (mod9)

 Em 27 de maio de 2015 09:52, Pedro José petroc...@gmail.com escreveu:

 Bom dia!

 Douglas,
 há valores ímpares de x que atendem 7^x≡ 4 (mod9)

 7^2 ≡ 4 (mod9) == x ≡  2 (mod3)

 7^1 ≡ 7 (mod9)
 7^2 ≡4 (mod9)
 7^3 ≡ 3*7 ≡ 1 (mod9)
 == 7^(2+3k) ≡ 7^2*(7^3)^k ≡ 4 (mod9)


 -- Mensagem encaminhada --
 De: Douglas Oliveira de Lima profdouglaso.del...@gmail.com
 Data: 26 de maio de 2015 23:37
 Assunto: Re: [obm-l] inteiros positivos
 Para: obm-l@mat.puc-rio.br obm-l@mat.puc-rio.br



 Bom, é fácil ver que x=1 e y=1 satisfaz a equação, assim caso y seja
 maior ou igual a 2,
 teremos que 7^x=4 (mod 9), desta forma x=2 (mod 4), ou podemos dizer que
 x é par da forma 2k,
 logo 7^2k-3^y=4, (7^k+2)(7^k-2)=3^y, mas nao existem duas potências de 3
 cuja diferença vale 4.
 Assim só existe uma solução.

 Abraço.
 Douglas Oliveira

 Em 26 de maio de 2015 22:41, marcone augusto araújo borges 
 marconeborge...@hotmail.com escreveu:

 Determine todos os inteiros positivos x e y tais que 7^x - 3^y = 4

 --
 Esta mensagem foi verificada pelo sistema de antivírus e
 acredita-se estar livre de perigo.



 --
 Esta mensagem foi verificada pelo sistema de antivírus e
 acredita-se estar livre de perigo.



 --
 Esta mensagem foi verificada pelo sistema de antivírus e
 acredita-se estar livre de perigo.


 --
 Esta mensagem foi verificada pelo sistema de antivírus e
 acredita-se estar livre de perigo.




-- 
Abraços

oɾnɐɹɐ ǝp oıɔıɹnɐɯ

-- 
Esta mensagem foi verificada pelo sistema de antiv�rus e
 acredita-se estar livre de perigo.



Re: [obm-l] inteiros positivos

2015-05-27 Por tôpico Mauricio de Araujo
Pensei em algo assim:

7 = -1 mod4
3 = -1 mod4

para que 7^x - 3^y = 4 = x, y devem ter a mesma paridade. Então

caso 1 ambos pares

x = 2k e y = 2m (k,m inteiros positivos)

7^2k - 3^2m = 4 = (7^k - 3^m)(7^k + 3^m) = 4 não é possível pois o produto
é maior do que 4 (em função do segundo fator).

caso 2 ambos ímpares

x = 2k+1 e y = 2m+1 (k,m inteiros não negativos)

para k=m=0 temos uma solução.

quem continua?

Em 27 de maio de 2015 15:09, Mauricio de Araujo 
mauricio.de.ara...@gmail.com escreveu:

 Douglas, em certo momento da sua demonstração, você diz o seguinte:

 ...7^x=4 (mod 9), desta forma x=2 (mod 4)...

 Porém, a primeira equação é satisfeita, por exemplo, por x = 5  (7^5 - 4 é
 múltiplo de 9!!!) não sendo, portanto, x côngruo a 2 mod4...

 Estou errado na minha avaliação?

 Em 27 de maio de 2015 10:58, Douglas Oliveira de Lima 
 profdouglaso.del...@gmail.com escreveu:

 Pense que x só pode assumir 4 formas,  4k, 4k+1, 4k+2, 4k+3.
 Em 27/05/2015 10:05, Pedro José petroc...@gmail.com escreveu:

 Bom dia!

 7^3 ≡ 4*7 ≡ 1 (mod9) e não 7^3 ≡ 3*7 ≡ 1 (mod9)

 Em 27 de maio de 2015 09:52, Pedro José petroc...@gmail.com escreveu:

 Bom dia!

 Douglas,
 há valores ímpares de x que atendem 7^x≡ 4 (mod9)

 7^2 ≡ 4 (mod9) == x ≡  2 (mod3)

 7^1 ≡ 7 (mod9)
 7^2 ≡4 (mod9)
 7^3 ≡ 3*7 ≡ 1 (mod9)
 == 7^(2+3k) ≡ 7^2*(7^3)^k ≡ 4 (mod9)


 -- Mensagem encaminhada --
 De: Douglas Oliveira de Lima profdouglaso.del...@gmail.com
 Data: 26 de maio de 2015 23:37
 Assunto: Re: [obm-l] inteiros positivos
 Para: obm-l@mat.puc-rio.br obm-l@mat.puc-rio.br



 Bom, é fácil ver que x=1 e y=1 satisfaz a equação, assim caso y seja
 maior ou igual a 2,
 teremos que 7^x=4 (mod 9), desta forma x=2 (mod 4), ou podemos dizer
 que x é par da forma 2k,
 logo 7^2k-3^y=4, (7^k+2)(7^k-2)=3^y, mas nao existem duas potências de
 3 cuja diferença vale 4.
 Assim só existe uma solução.

 Abraço.
 Douglas Oliveira

 Em 26 de maio de 2015 22:41, marcone augusto araújo borges 
 marconeborge...@hotmail.com escreveu:

 Determine todos os inteiros positivos x e y tais que 7^x - 3^y = 4

 --
 Esta mensagem foi verificada pelo sistema de antivírus e
 acredita-se estar livre de perigo.



 --
 Esta mensagem foi verificada pelo sistema de antivírus e
 acredita-se estar livre de perigo.



 --
 Esta mensagem foi verificada pelo sistema de antivírus e
 acredita-se estar livre de perigo.


 --
 Esta mensagem foi verificada pelo sistema de antivírus e
 acredita-se estar livre de perigo.




 --
 Abraços

 oɾnɐɹɐ ǝp oıɔıɹnɐɯ




-- 
Abraços

oɾnɐɹɐ ǝp oıɔıɹnɐɯ

-- 
Esta mensagem foi verificada pelo sistema de antiv�rus e
 acredita-se estar livre de perigo.



[obm-l] Re: [obm-l] Funções geradoras

2015-03-30 Por tôpico Mauricio de Araujo
Olá Israel, coloquei um pdf em inglês sobre o assunto no link abaixo.Espero
que te atenda. É recheado de exemplos...

https://drive.google.com/file/d/0B-1sAhj7LSlyT1VwMkxGU3lvTkE/view?usp=sharing

Em 28 de março de 2015 09:07, Israel Meireles Chrisostomo 
israelmchrisost...@gmail.com escreveu:

 Alguém aí tem um material falando sobre funções geradoras?

 --
 Esta mensagem foi verificada pelo sistema de antivírus e
 acredita-se estar livre de perigo.




-- 
Abraços

oɾnɐɹɐ ǝp oıɔıɹnɐɯ

-- 
Esta mensagem foi verificada pelo sistema de antiv�rus e
 acredita-se estar livre de perigo.



Re: [obm-l] Probabilidade Random quadratic equations

2015-03-03 Por tôpico Mauricio de Araujo
eis o livro:
https://mega.co.nz/#!O5ElSAyI!LmCHjd1xcLfex6fpH8I7pnGplcejFi4nAQRojHYgBTI

Em 3 de março de 2015 18:59, Douglas Oliveira de Lima 
profdouglaso.del...@gmail.com escreveu:

 Acho que encontrei a questão original,  num livro do professor de
 matemática estatística Frederick Mosteller da universidade de Harvard
 publicado em 1965 com o título FIFA CHALLENGING PROBLEMS IN PROBABILITY
 questão número 50 inclusive existem mais problemas fantásticos,  estou
 lendo
 Douglas oliveira
 Em 03/03/2015 13:47, Ralph Teixeira ralp...@gmail.com escreveu:

 Impossivel responder sem que se de uma ideia da distribuicao de
 probabilidade atendidas por r e s

 (Eu sou o chato da lista que reclama que tem muito problema de
 probabilidade que nao tem enunciado preciso...)

 Uma possibilidade eh tomar r e s distribuidos uniformemente e
 independentemente no intervalo [-A,A], e entao tomar A-+Inf.

 Agora sim: para ter raiz real devemos ter s=r^2. No quadrado
 [-A,A]x[-A,A] do plano rs, a regiao ruim (sem raiz real) eh acima da
 parabola, cuja area eh

 Int [r=-raiz(A), r=raiz(A)] (A-r^2) dr = 2Araiz(A)-2Araiz(A)/3=4Araiz(A)/3

 (ok, usei aqui A1, que eh razoavel jah que vamos tomar A-+Inf)

 Entao a probabilidade ruim seria isto dividido por 4A^2, isto eh,
 numero/raiz(A), que vai para 0 quando A-+Inf.

 Assim, nesse sentido, a resposta eh a probabilidade de ter raiz real eh
 1 (o que NAO significa garantia de ter raiz real).

 Abraco, Ralph

 P.S.: Se esta resposta lhe parece estranha, experimente desenhar a
 parabola y=x^2 no quadrado [-10,10]x[-10,10] e observe a area que fique
 acima dela em comparacao com o quadrado todo. Agora aumente o quadrado para
 [-100,100]x[-100,100], depois 1000, depois 1, e veja o que acontece --
 a regiao acima da parabola fica proporcionalmente cada vez menor, e tende a
 0.

 2015-03-03 10:55 GMT-03:00 Douglas Oliveira de Lima 
 profdouglaso.del...@gmail.com:

 Olá caros amigos, recebi um problema esta semana, e gostaria de uma
 ajuda (se possível), dos senhores.
 Pesquisei esse problema na internet e achei algumas divergências de
 soluções.
 Eis o problema:
 Qual a probabilidade da equação do segundo grau x^2+2rx+s=0 ter raiz
 real?



 Agradeço desde já a ajuda.
 Douglas Oliveira.


 --
 Esta mensagem foi verificada pelo sistema de antivírus e
 acredita-se estar livre de perigo.



 --
 Esta mensagem foi verificada pelo sistema de antivírus e
 acredita-se estar livre de perigo.


 --
 Esta mensagem foi verificada pelo sistema de antivírus e
 acredita-se estar livre de perigo.




-- 
Abraços

oɾnɐɹɐ ǝp oıɔıɹnɐɯ

-- 
Esta mensagem foi verificada pelo sistema de antiv�rus e
 acredita-se estar livre de perigo.



[obm-l] Re: [obm-l] Dúvida Combinatória

2014-09-29 Por tôpico Mauricio de Araujo
Olá Jorge!!

vou dar apenas uma indicação de como acho que daria para chegar numa
resposta...

Observe a figura abaixo:

_U_U_U_U_

Coloquemos nas posições U os 3 franceses e o portugues. Temos 4! de
possibilidades para fazer isso.

Agora precisamos colocar os brasileiros na posições _, podendo ambos
ficarem juntos.

Caso 1) brasileiros ficam juntos: Comb(5,1) . 2! = 10 maneiras.
Caso 2) brasileiros ficam separados: Comb(5,2) . 2! = 20 maneiras.

Agora para cada caso acima temos de contar a maneiras de se colocar os 5
argentinos nas posições _ restantes...

abc.

2014-09-29 9:32 GMT-03:00 Jorge Paulino jorge...@yahoo.com.br:

 Num grupo de 11 pessoas, 2 são brasileiros, 5 são argentinos, 3 são
 franceses e 1 é português.
 Quantas permutações podemos formar com essas 11 pessoas, de modo que não
 haja brasileiro ao lado de argentino?

 Grato,

 Jorge

 --
 Esta mensagem foi verificada pelo sistema de antivírus e
 acredita-se estar livre de perigo.

 =
 Instruções para entrar na lista, sair da lista e usar a lista em
 http://www.mat.puc-rio.br/~obmlistas/obm-l.html
 =




-- 
Abraços

oɾnɐɹɐ ǝp oıɔıɹnɐɯ

-- 
Esta mensagem foi verificada pelo sistema de antiv�rus e
 acredita-se estar livre de perigo.



[obm-l] Re: [obm-l] Dúvida Combinatória

2014-09-29 Por tôpico Mauricio de Araujo
desculpe não tem erro algum... desconsidere o email imediatamente acima...

2014-09-29 22:02 GMT-03:00 Mauricio de Araujo mauricio.de.ara...@gmail.com
:

 tem um erro na maneira como abri os casos... descubra qual é...

 2014-09-29 21:54 GMT-03:00 Mauricio de Araujo 
 mauricio.de.ara...@gmail.com:

 Olá Jorge!!

 vou dar apenas uma indicação de como acho que daria para chegar numa
 resposta...

 Observe a figura abaixo:

 _U_U_U_U_

 Coloquemos nas posições U os 3 franceses e o portugues. Temos 4! de
 possibilidades para fazer isso.

 Agora precisamos colocar os brasileiros na posições _, podendo ambos
 ficarem juntos.

 Caso 1) brasileiros ficam juntos: Comb(5,1) . 2! = 10 maneiras.
 Caso 2) brasileiros ficam separados: Comb(5,2) . 2! = 20 maneiras.

 Agora para cada caso acima temos de contar a maneiras de se colocar os 5
 argentinos nas posições _ restantes...

 abc.

 2014-09-29 9:32 GMT-03:00 Jorge Paulino jorge...@yahoo.com.br:

 Num grupo de 11 pessoas, 2 são brasileiros, 5 são argentinos, 3 são
 franceses e 1 é português.
 Quantas permutações podemos formar com essas 11 pessoas, de modo que não
 haja brasileiro ao lado de argentino?

 Grato,

 Jorge

 --
 Esta mensagem foi verificada pelo sistema de antivírus e
 acredita-se estar livre de perigo.

 
 =
 Instruções para entrar na lista, sair da lista e usar a lista em
 http://www.mat.puc-rio.br/~obmlistas/obm-l.html
 
 =




 --
 Abraços

 oɾnɐɹɐ ǝp oıɔıɹnɐɯ




 --
 Abraços

 oɾnɐɹɐ ǝp oıɔıɹnɐɯ




-- 
Abraços

oɾnɐɹɐ ǝp oıɔıɹnɐɯ

-- 
Esta mensagem foi verificada pelo sistema de antiv�rus e
 acredita-se estar livre de perigo.



[obm-l] Re: [obm-l] RES: [obm-l] Re: [obm-l] Re: [obm-l] RES: [obm-l] Problema da Olimpíada de Matemática de Moscou

2014-09-05 Por tôpico Mauricio de Araujo
é o antigo noves fora que minha mãe usava...


2014-09-04 21:25 GMT-03:00 Albert Bouskela bousk...@ymail.com:

 Olá!



 Pois é! Problemas (equações) que envolvem um determinado número (natural)
 e a soma dos algarismos que o compõem, geralmente, são resolvidos através
 da propriedade mencionada pelo Ralph:



 S(x) = x (mod. 9)



 Ou, o que dá no mesmo, mas as vezes pode ser mais útil:



 “x” e S(x) deixam o mesmo resto na divisão por 9.



 Exemplo:



 Mostre que a soma da soma da soma (3 vezes) dos algarismos de 50^50 e
 770^770 são iguais.



 

 Albert Bouskelá

 bousk...@ymail.com



 *De:* owner-ob...@mat.puc-rio.br [mailto:owner-ob...@mat.puc-rio.br] *Em
 nome de *Ralph Teixeira
 *Enviada em:* quarta-feira, 3 de setembro de 2014 21:27
 *Para:* obm-l@mat.puc-rio.br
 *Assunto:* [obm-l] Re: [obm-l] Re: [obm-l] RES: [obm-l] Problema da
 Olimpíada de Matemática de Moscou



 Ah, eh verdade, dah para acelerar MUITO notando que:



 S(x) = x (mod 9)



 Entao x+S(x)+S(S(x)) = 3x (mod 9)



 Isto eh, x+S(x)+S(S(x)) eh sempre divisivel por 3 -- e portanto nunca pode
 ser 1993.



 Abraco,

  Ralph



 2014-09-03 19:42 GMT-03:00 Mauricio de Araujo 
 mauricio.de.ara...@gmail.com:

 não tem solução!! hehehe



 2014-09-03 19:07 GMT-03:00 Albert Bouskela bousk...@ymail.com:

 Olá!



 A melhor solução é pelo “cheiro”



 1) x1899 | 1899+S(1899)+SS(1899)=19351993

 2) x1959 | 1959+S(1959)+SS(1959)=19891993

 3) S≥16 (x=1960) e SS≥2 (S=20)

 4) x=1993-16-2=1975

 5) 1960=x=1975

 6) Agora é no braço…

 7) Mas há uma surpresa no final!



 

 Albert Bouskelá

 bousk...@ymail.com

 *De:* owner-ob...@mat.puc-rio.br [mailto:owner-ob...@mat.puc-rio.br] *Em
 nome de *Mauricio de Araujo
 *Enviada em:* quarta-feira, 3 de setembro de 2014 11:36
 *Para:* obm-l@mat.puc-rio.br
 *Assunto:* [obm-l] Problema da Olimpíada de Matemática de Moscou



 Seja S(x) a soma dos dígitos de um inteiro positivo x.



 Resolver: x + S(x) + S(S(x)) = 1993.



 --

 Abraços


 oɾnɐɹɐ ǝp oıɔıɹnɐɯ

 --
 Esta mensagem foi verificada pelo sistema de antivírus e
 acredita-se estar livre de perigo.




-- 
Abraços

oɾnɐɹɐ ǝp oıɔıɹnɐɯ

-- 
Esta mensagem foi verificada pelo sistema de antiv�rus e
 acredita-se estar livre de perigo.



[obm-l] Problema da Olimpiada de Matemática de Moscou

2014-09-03 Por tôpico Mauricio de Araujo
Seja S(x) a soma dos dígitos de um inteiro positivo x.

Resolver: x + S(x) + S(S(x)) = 1993.

-- 
Abraços

oɾnɐɹɐ ǝp oıɔıɹnɐɯ

-- 
Esta mensagem foi verificada pelo sistema de antiv�rus e
 acredita-se estar livre de perigo.



[obm-l] Re: [obm-l] RES: [obm-l] Problema da Olimpiada de Matemática de Moscou

2014-09-03 Por tôpico Mauricio de Araujo
não tem solução!! hehehe


2014-09-03 19:07 GMT-03:00 Albert Bouskela bousk...@ymail.com:

 Olá!



 A melhor solução é pelo “cheiro”



 1) x1899 | 1899+S(1899)+SS(1899)=19351993

 2) x1959 | 1959+S(1959)+SS(1959)=19891993

 3) S≥16 (x=1960) e SS≥2 (S=20)

 4) x≤1993-16-2=1975

 5) 1960≤x≤1975

 6) Agora é no braço…

 7) Mas há uma surpresa no final!


 --

 *Albert Bouskelá*

 bousk...@ymail.com



 *De:* owner-ob...@mat.puc-rio.br [mailto:owner-ob...@mat.puc-rio.br] *Em
 nome de *Mauricio de Araujo
 *Enviada em:* quarta-feira, 3 de setembro de 2014 11:36
 *Para:* obm-l@mat.puc-rio.br
 *Assunto:* [obm-l] Problema da Olimpiada de Matemática de Moscou



 Seja S(x) a soma dos dígitos de um inteiro positivo x.



 Resolver: x + S(x) + S(S(x)) = 1993.



 --

 Abraços


 oɾnɐɹɐ ǝp oıɔıɹnɐɯ




 --
 Esta mensagem foi verificada pelo sistema de antiv?s e
 acredita-se estar livre de perigo.


 --
 Esta mensagem foi verificada pelo sistema de antivírus e
 acredita-se estar livre de perigo.




-- 
Abraços

oɾnɐɹɐ ǝp oıɔıɹnɐɯ

-- 
Esta mensagem foi verificada pelo sistema de antiv�rus e
 acredita-se estar livre de perigo.



[obm-l] Re: [obm-l] Livros de Física Matemática e a Probabilidade

2014-08-27 Por tôpico Mauricio de Araujo
Admitindo que todos os livros são diferentes entre si temos:

total de maneiras de dispor 8 livros em uma prateleira = 8!
arranjos que nos interessam = 2.4!.4!

assim P = 2.4!.4! / 8! = 1/35.


2014-08-26 23:23 GMT-03:00 João Sousa starterm...@hotmail.com:

 Quatro livros de Matemática e quatro de Física serão arrumados aleatoriamente,
 um ao lado do outro, numa prateleira. Qual  a probabilidade de que os
 livros de Matemática fiquem todos juntos e os de Física também fiquem
 todos juntos?



 Obrigado pela ajuda.



 João


 --
 Esta mensagem foi verificada pelo sistema de antivírus e
 acredita-se estar livre de perigo.




-- 
Abraços

oɾnɐɹɐ ǝp oıɔıɹnɐɯ

-- 
Esta mensagem foi verificada pelo sistema de antiv�rus e
 acredita-se estar livre de perigo.



[obm-l] Re: [obm-l] Combinatória - escadas

2014-08-18 Por tôpico Mauricio de Araujo
​Pense assim, ele está no sexto degrau.. para se chegar ao sexto degrau ou
ele veio do quinto​, ou do quarto ou terceiro degrau...

assim, o total de maneiras de se chegar no sexto degrau, N(6) será igual a
N(5)+N(4)+N(3)...

N(3) = N(2)+N(1)+N(0) = 2+1+0 = 3
N(4) = N(3)+N(2)+N(1) = 3+2+1 = 6
N(5) = N(4)+N(3)+N(2) = 6+3+2 = 11

N(6) = 20...

acho que é isso..



2014-08-18 15:37 GMT-03:00 Marcos Xavier mccxav...@hotmail.com:

 s degraus para chegar em casa. Como tem a





-- 
Abraços

oɾnɐɹɐ ǝp oıɔıɹnɐɯ

-- 
Esta mensagem foi verificada pelo sistema de antiv�rus e
 acredita-se estar livre de perigo.



[obm-l] Re: [obm-l] Re: [obm-l] Re: [obm-l] Combinatória - escadas

2014-08-18 Por tôpico Mauricio de Araujo
tem razão!
abraços.


2014-08-18 18:29 GMT-03:00 Ralph Teixeira ralp...@gmail.com:

 Hmm... Mas N(0)=1, certo? Entao fico com:

 N(3) = N(2)+N(1)+N(0) = 2+1+1 = 4
 N(4) = N(3)+N(2)+N(1) = 4+2+1 = 7
 N(5) = N(4)+N(3)+N(2) = 7+4+2 = 13
 N(6) = 24

 A sequencia eh 1,1,2,4,7,13,24,44,81,... ou seja os numeros de Tribonacci
 https://oeis.org/A73, porque a OEIS eh genial!

 Abraco,
  Ralph

 2014-08-18 16:15 GMT-03:00 Mauricio de Araujo 
 mauricio.de.ara...@gmail.com:

  Pense assim, ele está no sexto degrau.. para se chegar ao sexto degrau ou
  ele veio do quinto, ou do quarto ou terceiro degrau...
 
  assim, o total de maneiras de se chegar no sexto degrau, N(6) será igual
 a
  N(5)+N(4)+N(3)...
 
  N(3) = N(2)+N(1)+N(0) = 2+1+0 = 3
  N(4) = N(3)+N(2)+N(1) = 3+2+1 = 6
  N(5) = N(4)+N(3)+N(2) = 6+3+2 = 11
 
  N(6) = 20...
 
  acho que é isso..
 
 
 
  2014-08-18 15:37 GMT-03:00 Marcos Xavier mccxav...@hotmail.com:

 
  s degraus para chegar em casa. Como tem a
 
 
 
 
 
  --
  Abraços
 
  oɾnɐɹɐ ǝp oıɔıɹnɐɯ

 
 
  --
  Esta mensagem foi verificada pelo sistema de antivírus e
  acredita-se estar livre de perigo.

 --
 Esta mensagem foi verificada pelo sistema de antivírus e
 acredita-se estar livre de perigo.




-- 
Abraços

oɾnɐɹɐ ǝp oıɔıɹnɐɯ

-- 
Esta mensagem foi verificada pelo sistema de antiv�rus e
 acredita-se estar livre de perigo.



[obm-l] Re: [obm-l] Mais duas questões excelentes de geometria!!!

2014-05-23 Por tôpico Mauricio de Araujo
​boa tarde!!

você ainda tem essa apostila? Se sim, poderia enviar uma cópia para mim?
levante os custos por favor.

M.​


2014-05-15 17:24 GMT-03:00 Douglas Oliveira de Lima 
profdouglaso.del...@gmail.com:

 Quando estava estudando para a prova do colégio naval em 1997, no colégio
 e curso tamandare da ilha do governador(Diretores Orozimbo e Oswaldo) me
 deparei com uma apostila em xerox escrita pelos professores Carlos Victor e
 Eduardo Mauro, que tinham provas resolvidas do colégio naval e no final da
 apostila tinham questões propostas por eles, e uma delas que gostei muito e
 fiz na época(sem uso de calculo)
  quero compartilhar com os senhores, e também outro problema numero 2 que
 ainda não consegui fazer mas estou tentando. A saber so fui conseguir a
 apostila no final do curso perto da prova assim continuei estudando ate que
 passei em primeiro lugar geral de matemática(nesta época um ano atras
 pensei que não passaria numa prova dessas nunca).

 PROBLEMA 1)(Proposta por Carlos Victos e Eduardo Mauro ) Dado um quarto de
 circulo AOB, de raio OA, prolonga-se o raio OA, e pelo ponto B, traca-se
 uma perpendicular ao raio OB.

 a) Tracar a este quarto de circulo uma tangente MN, tal que a área do
 trapézio BMNO seja igual a uma área dada m^2.
 b) Ache o minimo da area do trapezio.

 PROBLEMA 2)Seja um cubo de aresta a. Seja N um ponto na diagonal de uma
 face lateral, M um ponto no círculo que se encontra no plano da base com
 centro no centro da base e raio (5/12)a  Encontre o menor valor da medida
 do segmento MN.

 --
 Esta mensagem foi verificada pelo sistema de antivírus e
 acredita-se estar livre de perigo.




-- 
Abraços

oɾnɐɹɐ ǝp oıɔıɹnɐɯ

-- 
Esta mensagem foi verificada pelo sistema de antiv�rus e
 acredita-se estar livre de perigo.



Re: [obm-l] off topic - livro caronnet

2014-04-14 Por tôpico Mauricio de Araujo
olá, me passe o link por favor;


2014-04-14 17:35 GMT-03:00 regis barros regisgbar...@yahoo.com.br:

 Olá Pessoal
 No ano passado teve contato com todos os livros do caronnet e fiz um scan
 deles caso vocês queiram é só pedir que mando link para o email pessoal.

 Regis
   Em Segunda-feira, 14 de Abril de 2014 14:34, Sergio Lima 
 sergi...@smt.ufrj.br escreveu:
  oi Hermann,

 Procure na Estante Virtual. Acho que voce encontrarah lah.
 Os livros tem excelentes exercicios.

 Abraco,
 sergio


 2014-04-20 13:06 GMT-03:00 Hermann ilhadepaqu...@bol.com.br:

  OFF - TOPIC

 Meus amigos, gostaria de saber se alguém já viu os livros do caronnet
 volumes 1, 2, 4 e 5.

 Porque há anos eu procuro e nunca vi.

 E a coleção é boa, na opinião de vocês?

 abraços
 Hermann

 --
 Esta mensagem foi verificada pelo sistema de antivírus e
 acredita-se estar livre de perigo.



 --
 Esta mensagem foi verificada pelo sistema de antivírus e
 acredita-se estar livre de perigo.



 --
 Esta mensagem foi verificada pelo sistema de antivírus e
 acredita-se estar livre de perigo.




-- 
Abraços

oɾnɐɹɐ ǝp oıɔıɹnɐɯ
*momentos excepcionais pedem ações excepcionais.*
*Os cemitérios estão cheios de pessoas insubstituíveis em seus ofícios.*

-- 
Esta mensagem foi verificada pelo sistema de antiv�rus e
 acredita-se estar livre de perigo.



Re: [obm-l]

2014-03-21 Por tôpico Mauricio de Araujo
Instruções para entrar na lista, sair da lista e usar a lista em
http://www.mat.puc-rio.br/~obmlistas/obm-l.html


2014-03-21 19:09 GMT-03:00 rodrigo pires de araújo rodrigopo...@hotmail.com
:

 EU NÃO DESEJO MAIS RECEBER MENSAGENS!!!

 --
 Esta mensagem foi verificada pelo sistema de antivírus e
 acredita-se estar livre de perigo.




-- 
Abraços

oɾnɐɹɐ ǝp oıɔıɹnɐɯ
*momentos excepcionais pedem ações excepcionais.*
*Os cemitérios estão cheios de pessoas insubstituíveis em seus ofícios.*

-- 
Esta mensagem foi verificada pelo sistema de antiv�rus e
 acredita-se estar livre de perigo.



[obm-l] Re: [obm-l] Re: [obm-l] Re: [obm-l] Re: [obm-l] Re: [obm-l] Combinatória

2014-03-18 Por tôpico Mauricio de Araujo
Veja uma contagem dupla:

partindo de _H1_M1_H2_M2_H3_M3_H4_ = aí vc coloca a M4 na terceira posição
livre ficando: H1M1M4H2M2H3M3H4

partindo de _H1_M4_H2_M2_H3_M3_H4_ = aí vc coloca a M1 na segunda posição
livre ficando: H1M1M4H2M2H3M3H4

ou seja, vc chegou na mesma configuração de duas maneira diferentes...


2014-03-18 9:45 GMT-03:00 Fabio Silva cacar...@yahoo.com:

 Olá amigos,

 Ainda insisto. Pensemos nas oito possibilidades de escolher um lugar para
 aquela mulher. Após isto, devemos pensar em escolher quantas possibilidades
 de mulheres posso colocar na primeira posição posição, na segunda e assim
 sucessivamente. O que daria um total de 4!. O mesmo pensamento seria para
 os homens, sendo igual a 4!.
 Daí, não vi contagem dobrada. E o resultado seria apenas o produto mesmo:
 8.4!.4!=4608 possibilidades.

 Onde estaria a contagem em dobro?

 Um abraço

 Fabio MS


   On Monday, March 17, 2014 10:52 PM, Walter Tadeu Nogueira da Silveira 
 wtade...@gmail.com wrote:
   Obrigado a todos. E, sim, Leo, foi engano. Seria C(5,4) formas de
 escolher a posição dos homens.

 Abs


 Em 17 de março de 2014 21:06, Pacini Bores pacini.bo...@globo.comescreveu:

 Olá,
 Nas soluções do Kleber e do Fabio, devemos retirar 3.4!.4! ; pois como o
 Leonardo falou, entre os homens os 3.4!.4! foram contado duas vezes.

 Abraços

 Pacini


 Em 17 de março de 2014 20:35, Leonardo Maia lpm...@gmail.com escreveu:

 Vejo a razão com o Walter (apesar de um typo), e não com o Kleber.

 Enxergo dupla contagem na solução do Kleber. Notem os dois espaços ao
 redor da 1a. mulher entre as 3 já alocadas, por exemplo. Quando se contam
 as possíveis posições da 4a. mulher, essas duas posições já são
 consideradas entre as 8 possibilidades, correspondendo aos dois possíveis
 ordenamentos de duas mulheres que eventualmente fiquem juntas ali. Depois,
  DE NOVO esses dois possíveis ordenamentos são contados no 4! das
 mulheres. Overcounting!

 Na solução do Walter, os dois fatores 4! estão corretos e devem ser
 multiplicados pelo número de possíveis entrelaçamentos das filas de
 homens e mulheres, que é dado pelo número de soluções da equação x1 + x2 +
 x3 + x4 + x5 = 4 onde cada variável só pode valer 0 ou 1 (cada variável
 corresponde ao número de homens na posição de cada espaço _ na solução do
 Walter). São 5, e não C(5,2), tais soluções. O Walter deve ter pensado uma
 coisa e escrito outra, pois o 2880 que julgo correto resulta do 5.

 Saudações,
 Leo.


 On Monday, March 17, 2014, Kleber Bastos klebe...@gmail.com wrote:

 Pensei aqui o problema de uma forma diferente:
 Como os homens não podem ficar juntos, temos que ter pelo menos uma mulher
 entre dois homens. Então vamos colocar os 4 homens em fila, sempre com uma
 mulher enrte 2:
 H M H M H M H
  Para isso precisamos usar 3 mulheres. Isso é o mínimo que temos que ter.
 Mas ainda temos uma mulher para colocar na fila em qualquer lugar. Os
 lugares possíveis para essa última mulher são 8, onde vou colocar os traços:
 _ H _ M _ H _ M _ H _ M _ H _
  Então temos 8 maneiras diferentes de colocar a última mulher. Além
 disso, podemos trocar os homens de lugar entre si (que pode ser feito de P
 4 = 4! maneiras) e as mulheres de lugar enter si (que pode ser feito de P4 =
 4! maneiras).
 Portanto teremos:
 = 8 . 4! . 4!
 = 8 . 24 . 24= 4608

 Abraços, Kleber.
 Sent from my iPad

 On 17/03/2014, at 19:06, Walter Tadeu Nogueira da Silveira 
 wtade...@gmail.com wrote:

 Amigos,

 Na questão: De quantas maneiras podemos dispor 4 homens e 4 mulheres em
 uma fila, sem que dois homens fiquem juntos?

 Pensei em amarrar as mulheres e escolher posições onde os homens
 poderiam ocupar sem ficar dois juntos. Depois permutar homens e mulheres.

 _ M _ M _ M _ M _

 C(5,2). P4. P4 = 2880 formas diferentes.

 O gabarito da questão diz 4608. Mas não concordei com essa resposta.

 Alguém poderia ajudar. Muito obrigado.
 --
 Walter Tadeu Nogueira da Silveira


 --
 Esta mensagem foi verificada pelo sistema de antivírus e
 acredita-se estar livre de perigo.


 --
 Esta mensagem foi verificada pelo sistema de antivírus e
 acredita-se estar livre de perigo.


 --
 Esta mensagem foi verificada pelo sistema de antivírus e
 acredita-se estar livre de perigo.



 --
 Esta mensagem foi verificada pelo sistema de antivírus e
 acredita-se estar livre de perigo.




 --
 Walter Tadeu Nogueira da Silveira
 http://www.professorwaltertadeu.mat.br


 --
 Esta mensagem foi verificada pelo sistema de antivírus e
 acredita-se estar livre de perigo.



 --
 Esta mensagem foi verificada pelo sistema de antivírus e
 acredita-se estar livre de perigo.




-- 
Abraços

oɾnɐɹɐ ǝp oıɔıɹnɐɯ
*momentos excepcionais pedem ações excepcionais.*
*Os cemitérios estão cheios de pessoas insubstituíveis em seus ofícios.*

-- 
Esta mensagem foi verificada pelo sistema de antiv�rus e
 acredita-se estar livre de perigo.



Re: [obm-l] [off topic] Apostila Desenho Geometrico Prof Brandao

2014-02-20 Por tôpico Mauricio de Araujo
Sérgio,

As apostilas eram do curso Impacto do Rio, já há tempos falido... não me
recordo de ter visto nada na apostila original mencionando copyright... vou
verificar de novo... mas para ter certeza de que não estamos infringindo
nenhuma lei, o mais correto é fazer uma consulta a um advogado. Pelo menos
não foi o caso de auferirmos lucro com a disponibilização do material..

Entendo que o que não está proibido é permitido; se não houver nada
proibindo a divulgação do material então não estamos cometendo crime. Mas
realmente precisamos checar...


2014-02-20 15:47 GMT-03:00 Sergio Lima sergi...@smt.ufrj.br:

 Caros Colegas,

 Desculpem-me por retomar esse tema e faze-lo de forma ainda mais
 [off topic] que antes.

 Alguem poderia me dizer se a divulgacao desse material iria
 infringir alguma regra/lei/recomendacao relativa aos direitos autorais?

 Sei que isso estah longe de ser o tema desta lista,
 mas como isso tambem foi o assunto de tantas mensagens,
 eu pensei que alguem da lista saberia me esclarecer isto.

 Agradeco antecipadamente por qualquer informacao.

 Abraco,
 sergio


 --
 Esta mensagem foi verificada pelo sistema de antivírus e
 acredita-se estar livre de perigo.




-- 
Abraços

oɾnɐɹɐ ǝp oıɔıɹnɐɯ
*momentos excepcionais pedem ações excepcionais.*
*Os cemitérios estão cheios de pessoas insubstituíveis em seus ofícios.*

-- 
Esta mensagem foi verificada pelo sistema de antiv�rus e
 acredita-se estar livre de perigo.



[obm-l] Re: [obm-l] Re: [obm-l] Re: [obm-l] Re: [obm-l] Círculos tangentes

2014-02-16 Por tôpico Mauricio de Araujo
try here:

yakovenko.files.wordpress.com/2009/11/cr.pdf


2014-02-16 12:16 GMT-03:00 Vanderlei Nemitz vanderma...@gmail.com:

 Onde encontro?
 Em 16/02/2014 12:12, carwatbr carwa...@yahoo.com.br escreveu:

 What's Mathematics de Courant e Robins. Lá tem a construção.
 Abraços,
 Carlos Juiti Watanabe.


  Mensagem original 
 De : Vanderlei Nemitz
 Data:16/02/2014 10:48 (GMT-03:00)
 Para: obm-l@mat.puc-rio.br
 Assunto: [obm-l] Re: [obm-l] Re: [obm-l] Círculos tangentes

 Está relacionado com o teorema dos círculos de Descartes ou teorema das
 círculos que se beijão. Não encontrei em nenhum lugar como construir os
 círculos interno e externo.
 Em 16/02/2014 10:41, Hermann ilhadepaqu...@bol.com.br escreveu:

  Não seria a interseção de duas perpendiculares que passassem pelo
 ponto médio dos segmentos que ligam os centros?

 - Original Message -
 *From:* Vanderlei Nemitz vanderma...@gmail.com
 *To:* obm-l@mat.puc-rio.br
 *Sent:* Sunday, February 16, 2014 8:08 AM
 *Subject:* [obm-l] Círculos tangentes

 Como construir o círculo tangente exteriormente a 3 círculos, tangentes
 dois a dois? Não consigo determinar o centro desse círculo.

 Obrigado!

 --
 Esta mensagem foi verificada pelo sistema de antivírus e
 acredita-se estar livre de perigo.


 --
 Esta mensagem foi verificada pelo sistema de antivírus e
 acredita-se estar livre de perigo.


 --
 Esta mensagem foi verificada pelo sistema de antivírus e
 acredita-se estar livre de perigo.
 --
 Esta mensagem foi verificada pelo sistema de antivírus e
 acredita-se estar livre de perigo.


 --
 Esta mensagem foi verificada pelo sistema de antivírus e
 acredita-se estar livre de perigo.




-- 
Abraços

oɾnɐɹɐ ǝp oıɔıɹnɐɯ
*momentos excepcionais pedem ações excepcionais.*
*Os cemitérios estão cheios de pessoas insubstituíveis em seus ofícios.*

-- 
Esta mensagem foi verificada pelo sistema de antiv�rus e
 acredita-se estar livre de perigo.



[obm-l] Re: [obm-l] Combinatória 2014

2014-01-22 Por tôpico Mauricio de Araujo
Este é o problema de Lucas... existe uma demonstração dele no livro de
combinatória do Morgado (Análise Combinatória e Probabilidade)...


2014/1/14 Antonio Paschoal barz...@dglnet.com.br

  Olá.

 Se possível for gostaria de uma ajuda com o seguinte problema de
 combinatória:

 “ Seis casais estão sentados ao redor de uma mesa circular. Quantas são as
 distribuições nas quais há alternância de homem e mulher porém

 não há nenhum casal sentado lado a lado.”



 Me parece claro que o número de distribuições alternadas é dada por
 PC(6)=5! x 6! .

 Acho que agora há que utilizar o princípio da Inclusão-Exclusão para
 filtrar os casais pareados.

 Essa é parte difícil do problema.



 Agradeço qualquer ajuda.



 Um abraço.



 Antonio Paschoal






 --
 http://www.avast.com/

 Este email está limpo de vírus e malwares porque a proteção do avast!
 Antivírus http://www.avast.com/ está ativa.


 --
 Esta mensagem foi verificada pelo sistema de antivírus e
 acredita-se estar livre de perigo.




-- 
Abraços

oɾnɐɹɐ ǝp oıɔıɹnɐɯ
*momentos excepcionais pedem ações excepcionais.*
*Os cemitérios estão cheios de pessoas insubstituíveis em seus ofícios.*

-- 
Esta mensagem foi verificada pelo sistema de antiv�rus e
 acredita-se estar livre de perigo.



Re: [obm-l] Apostila de Desenho 2 Impacto OFF TOPIC

2013-12-10 Por tôpico Mauricio de Araujo
Apostila recebida, muito obrigado!!!


2013/12/4 Jefferson Franca jeffma...@yahoo.com.br

 Muito obrigado


   Em Terça-feira, 3 de Dezembro de 2013 17:36, jjun...@fazenda.ms.gov.br
 jjun...@fazenda.ms.gov.br escreveu:
  Prezados amigos,

 Não há necessidade de reembolso.
 Os valores gastos não foram elevados.
 Vocês serão úteis a vários e distintos jovens pelo país, e, assim, eu
 também, indiretamente...

 Fraternalmente, João.



 - Mensagem Original -
 De:
 obm-l@mat.puc-rio.br

 Para:
 obm-l@mat.puc-rio.br obm-l@mat.puc-rio.br
 Cópia:~e

 Enviado:
 Tue, 3 Dec 2013 11:57:08 -0800 (PST)
 Assunto:
 Re: [obm-l] Apostila de Desenho 2 Impacto OFF TOPIC


 Obrigado João pelo envio do material.
 Me informe o numero e agencia de sua conta para que eu possa lhe pagar os
 custos de xerox e correio (e mais tempo perdido para fazer o serviço).
 Mais uma vez agradeço e o que precisar é só avisar.
 Abraços
 Graciliano


   Em Terça-feira, 3 de Dezembro de 2013 15:43, Carlos Victor 
 victorcar...@globo.com escreveu:
  Obrigado João,

 Envie-me a sua conta bancária para depósito, ok ?

 Também posso lhe enviar o custo pelo correio.

 Agradeço e fico a disposição para o que precisares.

 Abraços

 Carlos Victor


 Em 1 de dezembro de 2013 17:54, jjun...@fazenda.ms.gov.br escreveu:

  Senhores:

 Ontem (sábado), por volta das 15h em Campo Grande, foi enviada cópia da
 Apostila 2 de Desenho do IMPACTO, uma ao senhor Carlos Victor (Nilópolis -
 RJ), e outra a Graciliano Antônio Damazo (Penápolis - SP).

  ATT.
 João (Campo Grande - MS)

 --
 Esta mensagem foi verificada pelo sistema de antivírus e
 acredita-se estar livre de perigo.



 --
 Esta mensagem foi verificada pelo sistema de antivírus e

 acredita-se estar livre de perigo.



 --
 Esta mensagem foi verificada pelo sistema de antivírus e
 acredita-se estar livre de perigo.


 --
 Esta mensagem foi verificada pelo sistema de antiv�us e
 acredita-se estar livre de perigo.



 --
 Esta mensagem foi verificada pelo sistema de antivírus e
 acredita-se estar livre de perigo.




-- 
Abraços

oɾnɐɹɐ ǝp oıɔıɹnɐɯ
*momentos excepcionais pedem ações excepcionais.*
*Os cemitérios estão cheios de pessoas insubstituíveis em seus ofícios.*

-- 
Esta mensagem foi verificada pelo sistema de antiv�rus e
 acredita-se estar livre de perigo.



Re: [obm-l] Apostila de Desenho 2 Impacto OFF TOPIC

2013-11-30 Por tôpico Mauricio de Araujo
João, faço absoluta questão de reembolsar os custos... não tem sentido você
assumir isso... por favor, envie os dados bancários para eu depositar o
valor referente à minha parte.

Abraços.


2013/11/30 jjun...@fazenda.ms.gov.br

 Senhores:

 1) Ontem, ainda antes do meio em Campo Grande - MS, foi enviada a apostila
 para Maurício de Araújo (Uberlândia - MG), mesmo com a escrita sem acentos
 da atendente no envelope.

 2) Ela (a atendente) afirmou que o CEP de Jefferson França (66095-100) de
 Belém - PA, não existe, e estranhou (assim com eu) que o número viesse após
 ao que parece ser o bairro:  (...) São Miguel, 03 (...).

 3) Eram os únicos endereços que eu tinha até ontem.

 4) Ao que parece, outros também têm interesse na apostila, seria um
 presente a todos... mas, se muitos passarem a desejá-la, estou pensando em
 repassar o custo de R$ 15,00 a cada um, conforme alguns me já têm
 perguntado. R$ 9,00 referem-se à cópia, R$ 6,00 a postagem registrada.

 5) Hoje, se possível, enviarei as outras. Também, tirarei novas cópias.

 ATT.
 João




 --
 Esta mensagem foi verificada pelo sistema de antivírus e
 acredita-se estar livre de perigo.




-- 
Abraços

oɾnɐɹɐ ǝp oıɔıɹnɐɯ
*momentos excepcionais pedem ações excepcionais.*
*Os cemitérios estão cheios de pessoas insubstituíveis em seus ofícios.*

-- 
Esta mensagem foi verificada pelo sistema de antiv�rus e
 acredita-se estar livre de perigo.



[obm-l] Re: [obm-l] Exercício de Probabilidade

2013-11-27 Por tôpico Mauricio de Araujo
A probabilidade de uma ser perfeita e outra ser defeituosa é igual a
(3/8).(3/5) + (5/8).(2/5) = 19/40.

Então se a defeituosa vem da caixa A (prob = 3/8) e a perfeita vem da caixa
B (prob = 3/5) a probabilidade desejada será

(3/8).(3/5) / (19/40) = 9/19 = 47,4%


2013/11/27 Mauricio Barbosa oliho...@gmail.com

 Alguém poderia me ajudar no seguinte exercício:
 Uma caixa A contém 8 peças, das quais 3 são defeituosas e uma caixa B
 contém 5 peças, das quais 2 são defeituosas.  Uma peça é retirada
 aleatoriamente de cada caixa.  Se uma peça é defeituosa e a outra não, qual
 é a probabilidade de que a peça defeituosa venha da caixa A?
 Agradeço a ajuda!!!

 --
 Esta mensagem foi verificada pelo sistema de antivírus e
 acredita-se estar livre de perigo.




-- 
Abraços

oɾnɐɹɐ ǝp oıɔıɹnɐɯ
*momentos excepcionais pedem ações excepcionais.*
*Os cemitérios estão cheios de pessoas insubstituíveis em seus ofícios.*

-- 
Esta mensagem foi verificada pelo sistema de antiv�rus e
 acredita-se estar livre de perigo.



[obm-l] Re: [obm-l] Apostilas de desenho geométrico do prof Brandão

2013-11-01 Por tôpico Mauricio de Araujo
Prof. Renato, bom dia!!

Caso tenha dificuldades em escaná-las, eu pago os custos das cópias (xerox)
e do envio pelo correio. Assim que o material chegar a mim, eu peço para a
minha auxiliar escanear e repassar para os interessados.

Grato pela prontificação em disponibilizar o material.

Mauricio.


2013/10/10 Prof Renato Madeira profrenatomade...@gmail.com

 Eu tenho as originais em papel. Vou tentar escaneá-las e mando.

 Att, Renato Madeira.

 Em 10/10/2013, às 21:19, Mauricio de Araujo mauricio.de.ara...@gmail.com
 escreveu:

 Boa noite a todos.

 Gostaria de saber se alguém tem as apostilas de desenho geométrico do
 prof. Brandão do colégio Impacto do RJ (aquele do prof. Roquete)... Estudei
 lá em 1989 e o Brandão foi um professor inspirador.

 Quem tiver as as apostilas em pdf e puder disponibilizá-las ou souber quem
 poderia vendê-las para mim, agradeceria muito.

 --
 Abraços

 oɾnɐɹɐ ǝp oıɔıɹnɐɯ
 *momentos excepcionais pedem ações excepcionais.*
 *Os cemitérios estão cheios de pessoas insubstituíveis em seus ofícios.*

 --
 Esta mensagem foi verificada pelo sistema de antiv�rus e
 acredita-se estar livre de perigo.


 --
 Esta mensagem foi verificada pelo sistema de antivírus e
 acredita-se estar livre de perigo.




-- 
Abraços

oɾnɐɹɐ ǝp oıɔıɹnɐɯ
*momentos excepcionais pedem ações excepcionais.*
*Os cemitérios estão cheios de pessoas insubstituíveis em seus ofícios.*

-- 
Esta mensagem foi verificada pelo sistema de antiv�rus e
 acredita-se estar livre de perigo.



[obm-l] Re: [obm-l] Re: [obm-l] Re: [obm-l] Apostilas de desenho geométrico do prof Brandão

2013-11-01 Por tôpico Mauricio de Araujo
2013/11/1 tiago Santos tiago...@gmail.com

 Virgílio de Athayde


Tive aulas de Geometrica Descritiva com o professor Virgílio... mas não
conheço material de desenho geométrico que ele tenha feito​... se estiver
em formato digital agradeceria se disponibilizasse...





-- 
Abraços

oɾnɐɹɐ ǝp oıɔıɹnɐɯ
*momentos excepcionais pedem ações excepcionais.*
*Os cemitérios estão cheios de pessoas insubstituíveis em seus ofícios.*

-- 
Esta mensagem foi verificada pelo sistema de antiv�rus e
 acredita-se estar livre de perigo.



[obm-l] Re: [obm-l] Questão do IME

2013-10-29 Por tôpico Mauricio de Araujo
veja a solução em

https://www.dropbox.com/s/3wpkb4ht01oidsz/foto%205.PNG


2013/10/29 marcone augusto araújo borges marconeborge...@hotmail.com

 Sejam x = 370370370...37(89 algarismos) e y = 111...1000...0(30 algarismos
 1 e 30 algarismos 0)
 Calcule (x - y)^1/3

 --
 Esta mensagem foi verificada pelo sistema de antivírus e
 acredita-se estar livre de perigo.




-- 
Abraços

oɾnɐɹɐ ǝp oıɔıɹnɐɯ
*momentos excepcionais pedem ações excepcionais.*
*Os cemitérios estão cheios de pessoas insubstituíveis em seus ofícios.*

-- 
Esta mensagem foi verificada pelo sistema de antiv�rus e
 acredita-se estar livre de perigo.



[obm-l] Re: [obm-l] Re: [obm-l] Re: [obm-l] Re: [obm-l] Questão do IME

2013-10-29 Por tôpico Mauricio de Araujo
O IME tirou 4 questões da prova deste ano do referido livro do Suprun,
Ipsis litteris...

E a VestSeller aproveita para fazer a propaganda do livro pelo qual eles
cobram o olho da cara... pelo menos no marketing eles são bons (o dono é
engenheiro do ITA)...

e o povo do IME vai e ainda ajuda a valorizar o produto (que é bom
mesmo)... podiam ao menos dar uma alterada nos números da questão, sei lá...

De qq forma o livro está à venda (na verdade está em falta) na
Vestseller... veja o link

http://www.vestseller.com.br/detalhamento.asp?produto_id=471






2013/10/29 Hermann ilhadepaqu...@bol.com.br

 **
 O autor é Suprun tem hoje a venda por um absurdo de preço na
 editora Vestseller

 - Original Message -
 *From:* Vanderlei Nemitz vanderma...@gmail.com
 *To:* obm-l@mat.puc-rio.br
 *Sent:* Tuesday, October 29, 2013 4:13 PM
 *Subject:* [obm-l] Re: [obm-l] Re: [obm-l] Questão do IME

  Maurício:

 Que livro é esse? O IME retirou a questão na íntegra.


 Obrigado!


 Em 29 de outubro de 2013 15:56, Mauricio de Araujo 
 mauricio.de.ara...@gmail.com escreveu:

  veja a solução em

 https://www.dropbox.com/s/3wpkb4ht01oidsz/foto%205.PNG


 2013/10/29 marcone augusto araújo borges marconeborge...@hotmail.com

  Sejam x = 370370370...37(89 algarismos) e y = 111...1000...0(30
 algarismos 1 e 30 algarismos 0)
   Calcule (x - y)^1/3

 --
 Esta mensagem foi verificada pelo sistema de antivírus e
 acredita-se estar livre de perigo.




 --
  Abraços

 oɾnɐɹɐ ǝp oıɔıɹnɐɯ
 *momentos excepcionais pedem ações excepcionais.*
 *Os cemitérios estão cheios de pessoas insubstituíveis em seus ofícios.*

 --
 Esta mensagem foi verificada pelo sistema de antivírus e
 acredita-se estar livre de perigo.



 --
 Esta mensagem foi verificada pelo sistema de antiv�rus e
 acredita-se estar livre de perigo.


 --
 Esta mensagem foi verificada pelo sistema de antivírus e
 acredita-se estar livre de perigo.




-- 
Abraços

oɾnɐɹɐ ǝp oıɔıɹnɐɯ
*momentos excepcionais pedem ações excepcionais.*
*Os cemitérios estão cheios de pessoas insubstituíveis em seus ofícios.*

-- 
Esta mensagem foi verificada pelo sistema de antiv�rus e
 acredita-se estar livre de perigo.



[obm-l] Re: [obm-l] Re: [obm-l] Re: [obm-l] Re: [obm-l] Apostilas de desenho geométrico do prof Brandão

2013-10-14 Por tôpico Mauricio de Araujo
Olá!!

Me formei no Ita em 95... Estudei no Impacto com o Herndl, Fernando
Miglorância, os irmãos Marcus Torres dentre outros, inclusive do Santoro...
rotina pesada aquelas dos TDs, aulas aos sábados e simulados aos domingos
pela manhã sempre depois dos intermináveis discursos do Roquete... mas acho
que valeu à pena... Aquela escola tinha excelentes professores... pena que
tudo no mundo acaba... ou será que a graça é exatamente essa?!

Abraços.


2013/10/14 jjun...@fazenda.ms.gov.br

 Também estudei no Impacto em 89. Eu era conhecido por Belém, minha cidade
 natal, eu era da turma 1, formei-me no IME em 94.
 Faça-me recordar de ti, Maurício de Araújo. Não me lembrando de tua
 fisionomia. Lembro-me do Santoro, Longuinho, Vasconcelos, Ramos, Lyra,
 Hilton.
 Vou procurar essas apostilas, não sei se ainda as tenho.
 Caso as tenha, enviá-las-ei.



 - Mensagem Original -
 De:
 obm-l@mat.puc-rio.br

 Para:
 obm-l@mat.puc-rio.br obm-l@mat.puc-rio.br
 Cópia:

 Enviado:
 Mon, 14 Oct 2013 12:45:46 -0700 (PDT)
 Assunto:
 [obm-l] Re: [obm-l] Re: [obm-l] Apostilas de desenho geométrico do prof
 Brandão



 Me incluo no lista dos que gostariam de receber o material.
 Grato,
 Graciliano.
   --
 *De:* Jeferson Almir jefersonram...@gmail.com
 *Para:* obm-l@mat.puc-rio.br obm-l@mat.puc-rio.br
 *Enviadas:* Domingo, 13 de Outubro de 2013 12:29
 *Assunto:* [obm-l] Re: [obm-l] Apostilas de desenho geométrico do prof
 Brandão

 Se não for incômodo e for possível , eu também gostaria de receber o
 material.

 Em domingo, 13 de outubro de 2013, Marcelo Gomes escreveu:

 Olá professores da lista e professor Renato, bom dia.

 Se não for incômodo e for possível, também gostaria de receber o material.

 Desde já obrigado.

 Abraços, Marcelo.



 Em 10 de outubro de 2013 22:28, Prof Renato Madeira 
 profrenatomade...@gmail.com escreveu:

 Eu tenho as originais em papel. Vou tentar escaneá-las e mando.

 Att, Renato Madeira.

 Em 10/10/2013, às 21:19, Mauricio de Araujo mauricio.de.ara...@gmail.com
 escreveu:

  Boa noite a todos.

 Gostaria de saber se alguém tem as apostilas de desenho geométrico do
 prof. Brandão do colégio Impacto do RJ (aquele do prof. Roquete)... Estudei
 lá em 1989 e o Brandão foi um professor inspirador.

 Quem tiver as as apostilas em pdf e puder disponibilizá-las ou souber quem
 poderia vendê-las para mim, agradeceria muito.

 --
 Abraços

 oɾnɐɹɐ ǝp oıɔıɹnɐɯ
 *momentos excepcionais pedem ações excepcionais.*
 *Os cemitérios estão cheios de pessoas insubstituíveis em seus ofícios.*

 --
 Esta mensagem foi verificada pelo sistema de antiv�rus e
 acredita-se estar livre de perigo.


 --
 Esta mensagem foi verificada pelo sistema de antivírus e
 acredita-se estar livre de perigo.



 --
 Esta mensagem foi verificada pelo sistema de antivírus e
 acredita-se estar livre de perigo.


 --
 Esta mensagem foi verificada pelo sistema de antiv�us e
 acredita-se estar livre de perigo.



 --
 Esta mensagem foi verificada pelo sistema de antivrus e
 acredita-se estar livre de perigo.


 --
 Esta mensagem foi verificada pelo sistema de antivírus e
 acredita-se estar livre de perigo.




-- 
Abraços

oɾnɐɹɐ ǝp oıɔıɹnɐɯ
*momentos excepcionais pedem ações excepcionais.*
*Os cemitérios estão cheios de pessoas insubstituíveis em seus ofícios.*

-- 
Esta mensagem foi verificada pelo sistema de antiv�rus e
 acredita-se estar livre de perigo.



[obm-l] Apostilas de desenho geométrico do prof Brandão

2013-10-10 Por tôpico Mauricio de Araujo
Boa noite a todos.

Gostaria de saber se alguém tem as apostilas de desenho geométrico do prof.
Brandão do colégio Impacto do RJ (aquele do prof. Roquete)... Estudei lá em
1989 e o Brandão foi um professor inspirador.

Quem tiver as as apostilas em pdf e puder disponibilizá-las ou souber quem
poderia vendê-las para mim, agradeceria muito.

-- 
Abraços

oɾnɐɹɐ ǝp oıɔıɹnɐɯ
*momentos excepcionais pedem ações excepcionais.*
*Os cemitérios estão cheios de pessoas insubstituíveis em seus ofícios.*

-- 
Esta mensagem foi verificada pelo sistema de antiv�rus e
 acredita-se estar livre de perigo.



[obm-l] Re: [obm-l] Apostilas de desenho geométrico do prof Brandão

2013-10-10 Por tôpico Mauricio de Araujo
eu nem sei como agradecer... obrigado mesmo!!


2013/10/10 Prof Renato Madeira profrenatomade...@gmail.com

 Eu tenho as originais em papel. Vou tentar escaneá-las e mando.

 Att, Renato Madeira.

 Em 10/10/2013, às 21:19, Mauricio de Araujo mauricio.de.ara...@gmail.com
 escreveu:

 Boa noite a todos.

 Gostaria de saber se alguém tem as apostilas de desenho geométrico do
 prof. Brandão do colégio Impacto do RJ (aquele do prof. Roquete)... Estudei
 lá em 1989 e o Brandão foi um professor inspirador.

 Quem tiver as as apostilas em pdf e puder disponibilizá-las ou souber quem
 poderia vendê-las para mim, agradeceria muito.

 --
 Abraços

 oɾnɐɹɐ ǝp oıɔıɹnɐɯ
 *momentos excepcionais pedem ações excepcionais.*
 *Os cemitérios estão cheios de pessoas insubstituíveis em seus ofícios.*

 --
 Esta mensagem foi verificada pelo sistema de antiv�rus e
 acredita-se estar livre de perigo.


 --
 Esta mensagem foi verificada pelo sistema de antivírus e
 acredita-se estar livre de perigo.




-- 
Abraços

oɾnɐɹɐ ǝp oıɔıɹnɐɯ
*momentos excepcionais pedem ações excepcionais.*
*Os cemitérios estão cheios de pessoas insubstituíveis em seus ofícios.*

-- 
Esta mensagem foi verificada pelo sistema de antiv�rus e
 acredita-se estar livre de perigo.



[obm-l] Re: [obm-l] Apostilas de desenho geométrico do prof Brandão

2013-10-10 Por tôpico Mauricio de Araujo
se quiser xerocar e enviar por correio eu pago os custos!!!


2013/10/10 Mauricio de Araujo mauricio.de.ara...@gmail.com

 eu nem sei como agradecer... obrigado mesmo!!


 2013/10/10 Prof Renato Madeira profrenatomade...@gmail.com

 Eu tenho as originais em papel. Vou tentar escaneá-las e mando.

 Att, Renato Madeira.

 Em 10/10/2013, às 21:19, Mauricio de Araujo mauricio.de.ara...@gmail.com
 escreveu:

 Boa noite a todos.

 Gostaria de saber se alguém tem as apostilas de desenho geométrico do
 prof. Brandão do colégio Impacto do RJ (aquele do prof. Roquete)... Estudei
 lá em 1989 e o Brandão foi um professor inspirador.

 Quem tiver as as apostilas em pdf e puder disponibilizá-las ou souber
 quem poderia vendê-las para mim, agradeceria muito.

 --
 Abraços

 oɾnɐɹɐ ǝp oıɔıɹnɐɯ
 *momentos excepcionais pedem ações excepcionais.*
 *Os cemitérios estão cheios de pessoas insubstituíveis em seus ofícios.*

 --
 Esta mensagem foi verificada pelo sistema de antiv�rus e
 acredita-se estar livre de perigo.


 --
 Esta mensagem foi verificada pelo sistema de antivírus e
 acredita-se estar livre de perigo.




 --
 Abraços

 oɾnɐɹɐ ǝp oıɔıɹnɐɯ
 *momentos excepcionais pedem ações excepcionais.*
 *Os cemitérios estão cheios de pessoas insubstituíveis em seus ofícios.*




-- 
Abraços

oɾnɐɹɐ ǝp oıɔıɹnɐɯ
*momentos excepcionais pedem ações excepcionais.*
*Os cemitérios estão cheios de pessoas insubstituíveis em seus ofícios.*

-- 
Esta mensagem foi verificada pelo sistema de antiv�rus e
 acredita-se estar livre de perigo.



[obm-l] Problema de lógica

2013-08-17 Por tôpico Mauricio de Araujo
Eu disponho de três cartas de baralho, dois ases e um valete, e as disponho
sobre uma mesa com as faces voltadas para baixo, uma ao lado da outra.
Antes de virar as faces, eu anotei a posição de cada uma das cartas, de
maneira que eu sei onde os ases e o valete estão.
Tua missão é identificar um dos ases me fazendo apenas uma pergunta que
admita apenas SIM-NÃO como resposta. Para isso, você deverá apontar para
uma das cartas. Se a carta para a qual você apontar for um ás, a resposta à
tua pergunta será sincera (eu não mentirei); se a carta for um valete, a
resposta poderá ser sim ou não, aleatoriamente.

-- 
Abraços

oɾnɐɹɐ ǝp oıɔıɹnɐɯ
*momentos excepcionais pedem ações excepcionais.*
*Os cemitérios estão cheios de pessoas insubstituíveis em seus ofícios.*

-- 
Esta mensagem foi verificada pelo sistema de antiv�rus e
 acredita-se estar livre de perigo.



[obm-l] Re: [obm-l] Problema de lógica

2013-08-17 Por tôpico Mauricio de Araujo
Sim, a pergunta não precisa ter relação com a carta para a qual você
apontou... esta apenas vai orientar se a resposta vai ser sincera ou
aleatória...


2013/8/17 Johann Dirichlet peterdirich...@gmail.com

  Em 17-08-2013 21:24, Mauricio de Araujo escreveu:

  Eu disponho de três cartas de baralho, dois ases e um valete, e as
 disponho sobre uma mesa com as faces voltadas para baixo, uma ao lado da
 outra. Antes de virar as faces, eu anotei a posição de cada uma das cartas,
 de maneira que eu sei onde os ases e o valete estão.
 Tua missão é identificar um dos ases me fazendo apenas uma pergunta que
 admita apenas SIM-NÃO como resposta. Para isso, você deverá apontar para
 uma das cartas. Se a carta para a qual você apontar for um ás, a resposta à
 tua pergunta será sincera (eu não mentirei); se a carta for um valete, a
 resposta poderá ser sim ou não, aleatoriamente.


 O apontar para a carta é independente da pergunta, certo? No seguinte
 sentido: eu aponto para a carta do meio, mas pergunto se a primeira carta é
 um valete. Pode ser?

  --
  Abraços

 oɾnɐɹɐ ǝp oıɔıɹnɐɯ
 *momentos excepcionais pedem ações excepcionais.*
  *Os cemitérios estão cheios de pessoas insubstituíveis em seus ofícios.*

 --
 Esta mensagem foi verificada pelo sistema de antiv�rus e
 acredita-se estar livre de perigo.



 --
 Esta mensagem foi verificada pelo sistema de antivírus e
 acredita-se estar livre de perigo.




-- 
Abraços

oɾnɐɹɐ ǝp oıɔıɹnɐɯ
*momentos excepcionais pedem ações excepcionais.*
*Os cemitérios estão cheios de pessoas insubstituíveis em seus ofícios.*

-- 
Esta mensagem foi verificada pelo sistema de antiv�rus e
 acredita-se estar livre de perigo.



[obm-l] Enigma de Martin Gardner

2013-08-17 Por tôpico Mauricio de Araujo
Persistência de um número é o número de passos necessários para reduzi-lo a
um único dígito multiplicando todos os seus algarismos para obter um
segundo número, depois multiplicando todos os dígitos deste número para se
obter um terceiro número, e assim por diante, até que um número de um
dígito é obtido. Por exemplo, 77 tem uma persistência de quatro, porque
requer quatro etapas para reduzi-lo a um dígito: 77-49-36-18-8. O menor
número de persistência 1 é 10, o menor de persistência 2 é 25, o menor de
persistência 3 é 39, e o menor de persistência 4 é 77. Qual é o menor
número de persistência cinco?

-- 
Abraços

oɾnɐɹɐ ǝp oıɔıɹnɐɯ
*momentos excepcionais pedem ações excepcionais.*
*Os cemitérios estão cheios de pessoas insubstituíveis em seus ofícios.*

-- 
Esta mensagem foi verificada pelo sistema de antiv�rus e
 acredita-se estar livre de perigo.



[obm-l] Re: [obm-l] Re: [obm-l] Problema de lógica

2013-08-17 Por tôpico Mauricio de Araujo
P
​ara este problema a resposta é a seguinte:

Aponta para a carta do meio e pergunta: A carta da esquerda é um Ás?

se a carta do meio for um Ás, eu terei de falar a verdade... então você
escolhe a carta da esquerda ou da direita conforme a minha resposta seja
sim ou não, respectivamente...
se a carta do meio for o valete, então ambas as outras cartas serão ases, e
novamente a carta da esquerda será um ás se eu disser sim e a da direita
será ás se eu disser não...​


2013/8/17 Johann Dirichlet peterdirich...@gmail.com

  Em 17-08-2013 21:47, Mauricio de Araujo escreveu:

  Sim, a pergunta não precisa ter relação com a carta para a qual você
 apontou... esta apenas vai orientar se a resposta vai ser sincera ou
 aleatória...


 Então a resposta vai ter que, pelo menos, forçar o cabra a falar a verdade
 a qualquer custo. Eu me lembro de um problema parecido:

 Mulheres de óculos escuros respondiam perguntas. Se os olhos fossem azuis,
 elas falavam a verdade. Se fossem castanhos, mentira. Como saber a cor dos
 olhos de cada uma?

 A pergunta para fazer a elas era algo como Se eu te perguntasse qual a
 cor dos olhos de cada uma de vós, qual seria a sua resposta?.





 2013/8/17 Johann Dirichlet peterdirich...@gmail.com

  Em 17-08-2013 21:24, Mauricio de Araujo escreveu:

  Eu disponho de três cartas de baralho, dois ases e um valete, e as
 disponho sobre uma mesa com as faces voltadas para baixo, uma ao lado da
 outra. Antes de virar as faces, eu anotei a posição de cada uma das cartas,
 de maneira que eu sei onde os ases e o valete estão.
 Tua missão é identificar um dos ases me fazendo apenas uma pergunta que
 admita apenas SIM-NÃO como resposta. Para isso, você deverá apontar para
 uma das cartas. Se a carta para a qual você apontar for um ás, a resposta à
 tua pergunta será sincera (eu não mentirei); se a carta for um valete, a
 resposta poderá ser sim ou não, aleatoriamente.


  O apontar para a carta é independente da pergunta, certo? No seguinte
 sentido: eu aponto para a carta do meio, mas pergunto se a primeira carta é
 um valete. Pode ser?

  --
  Abraços

 oɾnɐɹɐ ǝp oıɔıɹnɐɯ
 *momentos excepcionais pedem ações excepcionais.*
  *Os cemitérios estão cheios de pessoas insubstituíveis em seus ofícios.*

  --
 Esta mensagem foi verificada pelo sistema de antiv�rus e
 acredita-se estar livre de perigo.



 --
 Esta mensagem foi verificada pelo sistema de antivírus e
 acredita-se estar livre de perigo.




  --
  Abraços

 oɾnɐɹɐ ǝp oıɔıɹnɐɯ
 *momentos excepcionais pedem ações excepcionais.*
  *Os cemitérios estão cheios de pessoas insubstituíveis em seus ofícios.*

 --
 Esta mensagem foi verificada pelo sistema de antiv�rus e
 acredita-se estar livre de perigo.



 --
 Esta mensagem foi verificada pelo sistema de antivírus e
 acredita-se estar livre de perigo.




-- 
Abraços

oɾnɐɹɐ ǝp oıɔıɹnɐɯ
*momentos excepcionais pedem ações excepcionais.*
*Os cemitérios estão cheios de pessoas insubstituíveis em seus ofícios.*

-- 
Esta mensagem foi verificada pelo sistema de antiv�rus e
 acredita-se estar livre de perigo.



[obm-l] Re: [obm-l] Combinatória

2013-08-04 Por tôpico Mauricio de Araujo
2013/8/3 marcone augusto araújo borges marconeborge...@hotmail.com

 000


​Acho que o gabarito está errado...

Você pode pensar assim, considerando uma correspondência:


1 o 1 o 1 o 1 o 1 o 1 corresponde ao número 111.111
1 1 1 o o o o o 1 1 1 corresponde ao número 300.003
o o 1 1 1 1 o o 1 1 o corresponde ao número 004020 ou 4.020
o o o o o 1 1 1 1 1 1 corresponde ao número 06 ou 6

Logo o que se quer será igual a permutação de 11 elementos onde 6 são
iguais a 1 e 5 são iguais a o...

N = 11!/(6!.5!) =  462 que é C(11,6)...

Se não cometi algum erro acho que é isso...

​



-- 
Abraços

oɾnɐɹɐ ǝp oıɔıɹnɐɯ
*momentos excepcionais pedem ações excepcionais.*
*Os cemitérios estão cheios de pessoas insubstituíveis em seus ofícios.*

-- 
Esta mensagem foi verificada pelo sistema de antiv�rus e
 acredita-se estar livre de perigo.



Re: [obm-l] Probabilidade - muito interessante...

2013-07-12 Por tôpico Mauricio de Araujo
O raciocínio é este mesmo!! Sempre que passo este problema para alguns
alunos, eles inicialmente se assustam pois acreditam que a solução é
complicada... então recomendo que pensem um pouco na dinâmica do embarque
para perceberem que a solução não é tão complicada assim...

[]


2013/7/11 Henrique Rennó henrique.re...@gmail.com

 Se a primeira pessoa sentar justamente no seu assento, todas as outras
 também sentarão corretamente porque já tem os cartões de embarque e
 encontrarão seus assentos disponíveis e a última pessoa encontrará seu
 assento disponível. Se a primeira pessoa sentar no assento que a última
 sentaria, todas as outras irão sentar corretamente e a última encontrará
 seu assento ocupado pela primeira sobrando apenas o assento da primeira. Se
 a primeira sentar em um assento que não seja o dela nem o da última pessoa,
 uma das outras pessoas irá encontrar seu assento ocupado pela primeira e
 sentará ou no assento da primeira (e a última encontrará seu assento
 disponível), ou da última (e a última encontrará seu assento ocupado) ou em
 outro assento e as possibilidades para a próxima que iria sentar neste
 assento seriam as mesmas da anterior. O número de possibilidades é sempre
 par onde metade deixa o último assento disponível e metade deixa ocupado.

 A solução está correta? Será que existe uma solução mais simples?

 2013/7/11 Mauricio de Araujo mauricio.de.ara...@gmail.com

 *Recentemente, eu peguei um avião que tinha 137 assentos. Eu gosto
 sempre de ser o primeiro a embarcar e não foi diferente nesta ocasião.
 Infelizmente, assim que eu entrei no avião, percebi que havia perdido o meu
 cartão de embarque e não conseguia me lembrar de qual era o meu assento.
 Sem saber o que fazer, eu escolhi aleatoriamente um assento qualquer e me
 sentei. Claro que havia a probabilidade de 1/137 de eu ter escolhido o
 assento correto, ou seja, aquele que estava marcado no meu cartão de
 embarque. À medida que os demais passageiros embarcavam, cada um se dirigia
 ao seu assento e sentava-se, a menos que o mesmo estivesse ocupado. Neste
 caso, o passageiro abria mão de sentar-se no assento que estava
 originalmente atribuído a ele (conforme o cartão de embarque) e escolhia um
 outro assento qualquer para se sentar. Percebi que fui o único passageiro
 que perdeu o cartão de embarque.*
 *
 *
 *A questão que se coloca é a seguinte: qual a probabilidade de o último
 passageiro a embarcar encontrar o seu assento desocupado, ou seja,
 encontrar o assento que está no seu cartão de embarque disponível para ele
 se sentar?*

 Este problema está explicado no livro Introduction to counting and
 probability do David Patrick e tem uma resposta surpreendente: a
 probabilidade é de 50%...

 Para sentir a solução, vale a pena pensar no problema para os casos em
 que o avião tem 2, 3, 4 e 5 assentos...

 --
 Abraços

 oɾnɐɹɐ ǝp oıɔıɹnɐɯ
 *momentos excepcionais pedem ações excepcionais.*
 *Os cemitérios estão cheios de pessoas insubstituíveis em seus ofícios.*

 --
 Esta mensagem foi verificada pelo sistema de antivírus e
 acredita-se estar livre de perigo.




 --
 Henrique

 --
 Esta mensagem foi verificada pelo sistema de antivírus e
 acredita-se estar livre de perigo.




-- 
Abraços

oɾnɐɹɐ ǝp oıɔıɹnɐɯ
*momentos excepcionais pedem ações excepcionais.*
*Os cemitérios estão cheios de pessoas insubstituíveis em seus ofícios.*

-- 
Esta mensagem foi verificada pelo sistema de antiv�rus e
 acredita-se estar livre de perigo.



Re: [obm-l] Probabilidade - muito interessante...

2013-07-12 Por tôpico Mauricio de Araujo
isso mesmo!


2013/7/11 Eduardo Wilner eduardowil...@yahoo.com.br

 Consideremos o embarque dos 136 passageiros, inclusive você, i.e.
 excluindo o último (consideramos o voo lotado)
 Assim que alguém (inclusive você) ocupar o seu lugar ou o do último
 passageiro a embarcar, os passageiros seguintes encontrarão o próprio lugar
 vago, ocupando-o.
 Portanto, quando o centésimo trigésimo sétimo passageiro embarcar,
 encontrará vago ou o próprio lugar ou o seu, com probabilidade meio à
 meio


   --
  *De:* Mauricio de Araujo mauricio.de.ara...@gmail.com
 *Para:* obm-l@mat.puc-rio.br
 *Enviadas:* Quinta-feira, 11 de Julho de 2013 10:16
 *Assunto:* [obm-l] Probabilidade - muito interessante...

 *Recentemente, eu peguei um avião que tinha 137 assentos. Eu gosto
 sempre de ser o primeiro a embarcar e não foi diferente nesta ocasião.
 Infelizmente, assim que eu entrei no avião, percebi que havia perdido o meu
 cartão de embarque e não conseguia me lembrar de qual era o meu assento.
 Sem saber o que fazer, eu escolhi aleatoriamente um assento qualquer e me
 sentei. Claro que havia a probabilidade de 1/137 de eu ter escolhido o
 assento correto, ou seja, aquele que estava marcado no meu cartão de
 embarque. À medida que os demais passageiros embarcavam, cada um se dirigia
 ao seu assento e sentava-se, a menos que o mesmo estivesse ocupado. Neste
 caso, o passageiro abria mão de sentar-se no assento que estava
 originalmente atribuído a ele (conforme o cartão de embarque) e escolhia um
 outro assento qualquer para se sentar. Percebi que fui o único passageiro
 que perdeu o cartão de embarque.*
 *
 *
 *A questão que se coloca é a seguinte: qual a probabilidade de o último
 passageiro a embarcar encontrar o seu assento desocupado, ou seja,
 encontrar o assento que está no seu cartão de embarque disponível para ele
 se sentar?*

 Este problema está explicado no livro Introduction to counting and
 probability do David Patrick e tem uma resposta surpreendente: a
 probabilidade é de 50%...

 Para sentir a solução, vale a pena pensar no problema para os casos em
 que o avião tem 2, 3, 4 e 5 assentos...

 --
 Abraços

 oɾnɐɹɐ ǝp oıɔıɹnɐɯ
 *momentos excepcionais pedem ações excepcionais.*
 *Os cemitérios estão cheios de pessoas insubstituíveis em seus ofícios.*

 --
 Esta mensagem foi verificada pelo sistema de antiv�us e
 acredita-se estar livre de perigo.



 --
 Esta mensagem foi verificada pelo sistema de antivírus e
 acredita-se estar livre de perigo.




-- 
Abraços

oɾnɐɹɐ ǝp oıɔıɹnɐɯ
*momentos excepcionais pedem ações excepcionais.*
*Os cemitérios estão cheios de pessoas insubstituíveis em seus ofícios.*

-- 
Esta mensagem foi verificada pelo sistema de antiv�rus e
 acredita-se estar livre de perigo.



  1   2   >